You are on page 1of 130

MOCK EXAMS CAF AUTUMN 2022

Financial Accounting and Reporting I


Date: 23rd August 2022
3 Hours - 100 Marks
Additional reading time 15 minutes
QUESTION PAPER

Question-1
Butt limited (BL) runs a cruise business across oceans. Following information in respect of one of BL’s cruise ships is
available:

(i) BL bought a cruise ship on 1 March 2018. After completing all the required formalities, the ship was ready
to sail on 1 April 2018.
(ii) Details regarding components of the ship are as under:

Estimated
Cost residual value
Component Useful life
(Rs. in million) (Rs. in million)
Engine 840 50,000 hours 40
Body 535 25 years 35
Dry-docking (overhaul) 60 5 years -

(iii) On 1 May 2019, the ship suffered an accident which damaged its body. Repair work took 2 months and
costed Rs. 26 million. The repair work did not change useful life and residual values of the components.

(iv) The average monthly sailing of the ship during the last three years are as under:

Year Hours
2018 360
2019 480
2020 600

(v) BL uses revaluation model for subsequent measurement. BL accounts for revaluation on net replacement
value method and transfers the maximum possible amount from the revaluation surplus to retained earnings on
an annual basis.

(vi) The revalued amounts of the ship as at 31 December 2019 and 2020 were determined as Rs. 1,400 million and
Rs. 1,000 million respectively. Revalued amounts are apportioned between the components on the basis of
their book values before the revaluation.

Required:
Prepare necessary journal entries to record the above transaction from the date of acquisition of the ship to the year
ended 31 December 2020. (17)
Question 2: You have recently joined as the finance minister of Nishat Limited. While reviewing the draft
Financial statements for the year ended 31st December 2020 prepared by the junior accountant, you have noted
the following:

i. QIn January 2020, Government allotted an industrial plot to NL at a prime location subject to the condition that NL
uwill establish a factory. NL constructed the factory building which was available for use on 1 October 2020. Due to
edelay in recruitment of key factory employees, the production activities will commence on 15 March 2021.
s
The accountant has not recorded the land as it was given free of cost. While the factory building is still appearing in capital
t
work in progress as production activities will commence on 15 March 2021.(03)
i
o
(ii) NL acquired a three-story building on 1 March 2020. NL uses the ground floor for its marketing department
n
while remaining two floors were in excess of NL’s need and therefore were rented out. The first floor was rented
out on 1 June 2020 and the second floor was rented out on 1 December 2020.
2
The accountant has recorded the building as property, plant and equipment. The depreciation on
ground, first and second floors has been computed from1 March 2020, 1 June 2020 and 1 December 2020
respectively. (03)

(iii) NL is constructing a power generation plant for its factory. The project started on1 February 2020 and would
complete on 30 November 2021. The work remained suspended for 3 months. The projectis financed through long
term loan, acquired specifically on 1 January 2020. The unutilized amount of loan is kept in a separate saving
account.

The accountant has deducted income of separate saving account from full year’s interest on loan and presented the
net amount as finance cost in the statement of profit or loss. (04)

The accounting policy of NL is to carry land and building at fair value (wherever permitted by IFRS).

Required:
Discuss how the above issues should be dealt in the financial statements of NL for the year Ended 31 December 2020 in
accordance with the requirements of IFRS.
QUESTION 3:
The draft balance sheet of Time Life Enterprises (TLE) as on December 31, 2008, is being prepared. On
reviewing the accounts of the business, its auditor found that the records have been correctly maintained
except for the following events:
(i) On January 17, 2008 a contract was signed for the purchase of a machine from Makers Limited for
Rs. 1,125,000 which is to be delivered on July 17, 2009. TLE paid an advance of Rs. 450,000 on the
signing of the contract and the balance was to be paid on delivery of the machine. The advance was
debited to plant and machinery account.
(ii) Installation of a machine was completed on January 21, 2008. The cost of machine of Rs.2,700,000
was debited to plant and machinery account. The cost of installation amounting to Rs. 300,000 had
been debited to Repairs Account.

The depreciation is charged on declining balance method at 10 per cent per annum. Depreciation on
additions is provided from the month in which the asset is acquired. Depreciation expenses for the year 2008
have been correctly calculated and recorded except for the impact of errors discussed above.

Required: Pass the adjusting entries. (08)

Question 4:
The following information pertains to the financial statements of Home Dynamics Limited (HDL), a listed
company, for the year ended 31 December 2016:
(i) Profit after tax for the year:
Rs. in million
Profit from continuing operations – net of tax 765
Profit from discontinued operations – net of tax 155
Profit after tax 920

(ii) Shareholders’ equity as on 1 January 2016 comprised of:


• 10 million ordinary shares of Rs. 10 each, having market value of Rs. 25 each.
• 4 million cumulative preference shares of Rs. 10 each entitled to a cumulative dividend at
10%.
(iii) On 31 March 2016, HDL announced 40% right shares to its ordinary shareholders at
Rs. 25 per share. The entitlement date of right shares was 31 May 2016. The market price per share
immediately before the announcement date and entitlement date was Rs. 28 and Rs. 32 respectively.
(iv) On 2 August 2016, HDL announced 20% bonus issue. The entitlement date of bonus shares was 31
August 2016.
(v) On 1 February 2017, the board of directors announced 20% cash dividend and 10% bonus issue
being the final dividend to the ordinary shareholders and 10% cash dividend for preference
shareholders.
Required:
Calculate basic earnings per share for inclusion in HDL’s financial statements for the year ended
31 December 2016. Show all relevant calculations. (10)
Question 5
Galaxy Brothers commenced their business on 1 January 2013 with cash of Rs. 50 million, a building valued at Rs. 25
million and a motor vehicle costing Rs. 1.4 million. Following is the summarized Trial Balance as of 31 December
2013:

Debit Credit
Particulars
Rs. in million
Sales 136.0
0
Cost of sales (including depreciation expense of Rs. 9 million) 83.50
Operating and selling expenses (including depreciation expense of Rs. 6.25 million) 37.30
Miscellaneous income (net of loss of Rs. 0.35 on settlement of total loss claim) 0.50
Finance charges 2.50
Taxation expense 6.00
Cash and bank balances 5.00
Bank overdraft 23.00
Accounts receivable 18.00
Provision for doubtful debts 0.90
Closing inventory 10.00
Accounts payable 14.00
Interest payable 1.20
Provision for taxation (net of payments) 1.00
Partners' capital (net of cash withdrawals) 73.95
12% Long term loan payable 25.00
Property, plant and equipment 128.25
Accumulated depreciation 15.00
290.55 290.55

Settlement of the insurance claim pertained to an accident of a new car costing Rs. 1.8 million and having a
depreciation charge of Rs. 0.25 million for the period in use.
Required:
Prepare a statement of cash flow for the year ended 31 December 2013. (18)
Question 6
Select the most appropriate answer from the options available for each of the following Multiple Choice Questions
(MCQs).

1.Which of the following future cash flows should NOT be included in the calculation of value in use of an
asset?

(a) Cash flows on maintaining the asset’s performance


(b) Cash flows on enhancing the asset’s performance
(c) Cash flows from continuing use of the asset
(d) Cash flows from disposal of the asset
(01)

2. When an impairment review is carried out, an impaired asset is measured at:

(a) fair value less cost to sell (b) value in use


(c) cost (d) recoverable amount (01)

3. Which of the following would be an external indicator that an asset of an entity may be
impaired?
(a) Increase in central bank discount rates
(b) Decline in economic performance of an asset
(c) Physical obsolescence of an asset
(d) Future restructuring plan of an asset

(01)
4. Which of the following is NOT a measurement base for assets as referred in the Conceptual Framework?

(a) Value in use (b) Fulfilment value


(c) Current cost (d) Fair value (01)

5. An NPO uses restrictive fund method to report its contributions revenue. The fund deferred balance) related
to shall not be presented in net asse statement of financial
position.
(a) Endowment contributions
(b) Restricted contributions reported in restricted fund
(c) Restricted contributions reported in general fund
(d) Unrestricted contributions reported in general fund (01)

6. Statement of income and expenditures is based on:


(a) Cash accounting
(b) Accrual accounting
(c) Government accounting
(d) Management accounting (01)
7. An entity made a profit of Rs. 550,000 for the year 2020 based on historical cost accounting
principles. It had opening capital of Rs. 1,500,000. During 2020, specific prices indices increased by
15% while general price indices increased by 10%. How much profit should be recorded for 2020
under physical capital maintenance concept?

(a) Rs. 325,000 (b) Rs. 400,000


(c) Rs. 467,500 (d) Rs. 495,000 (01)

8 In order to survive in the long run, a business must generate positive net cash flow from:

(a) investing activities


(b) operating activities
(c) financing activities
(d) both (a) and (b) (01)

Question 7
Following information pertains to Cocomo Limited (CL):

i. Shareholders' equity as on 1 January 2020:

Rs. in million
al (Rs. 100 each) 250
Share premium 138
Retained earnings 142
Revaluation surplus: Land 25
Buildings 20

ii. Profit and transfer of incremental depreciation as per the draft financial statements for the year ended 31
December 2020 amounted to Rs. 45 million and Rs. 5 million respectively.

iii. Dividends for the last two years:

For the year ended *Interim cash Final bonus dividend


dividend
31 December 2019 10% 20%
31 December 2020 12% 15%
*Declared with half yearly accounts

iv. CL uses revaluation model for subsequent measurement of its land and buildings only. The
revalued amounts of land and buildings have been assessed at 31 December 2020 but not
incorporated in draft financial statements. The relevant details are as under:

Land Building
s
--- Rs. in million ---
Balances as on 31 December 2020 before revaluation:
Cost 75 240
Accumulated depreciation - 60
Revalued amounts assessed at 31 December 2020 65 158

Required:
Prepare CL’s statement of changes in equity for the year ended 31 December 2020. (10)
(Column for total and comparative figures are not required)
Question 8:
The Mayfair Sports and Social Club's assets and liabilities for the previous and current year were as follows:
At December At December
31,2002 31, 2003
Rs. Rs.
Equipment 125,000 140,000
Subscriptions in arrears 10,000 9,000
Subscriptions in advance 6,500 5,500
Creditors for soft drinks stock 17,500 21,500
Soft Drinks stocks 40,000 30,000
Rent owing 7,500 5,000
Electricity owing 5,250 7,000
Bank balance 36,150 65,000

During the year the cash receipts were:

Subscriptions (including Rs. 3,000 of arrears from


previous year and the balance to be written off) 105,000
Soft Drinks takings 205,000
Sale of tickets for annual dinner 120,000
Sale of raffle tickets 9,000
The following payments were made during the year:
Affiliation fees 5,000
Purchase of equipment 40,000
Soft Drinks stocks 102,500
Soft Drinks salesman's wages 37,500
Catering 72,000
Hire of band 15,000
Raffle prizes 3,000
Rent of hall 75,000
Printing and postage 10,000
Electricity 29,050
Hon. Secretary's expenses 6,100
Repairs to equipment 15,000

Required: The Mayfair Sports and Social Club's


(i) Soft Drinks trading account for the year ended December 31, 2003 (03)
(ii) Income and expenditure account for the year ended Dec. 31, 2003 (07)
(iii) Balance sheet as at 31 December 2003. (04)

Question 9:

Discuss the limitations of Ratio Analysis (05)


Solution of Mock FAR I

Answer 1

General Journal Rs. in million


Date Description Debit Credit
01-03-2018 Cruise ship 1,435.00
0.5 marks Bank 1,435.00

31-12-2018 Depreciation expense (W-1) 75.84


0.5 marks Accumulated depreciation - Cruise ship 75.84

01-05-2019 Repair cost 26.00


0.5 marks Bank 26.00

31-12-2019 Depreciation expense (W-1) 108.80


0.5 marks Accumulated depreciation - Cruise ship 108.80

31-12-2019 Accumulated depreciation - Cruise ship 184.64


0.5 marks Cruise ship 75.84+108.80 184.64

31-12-2019 Cruise ship 149.64


1 marks Revaluation surplus 149.64

31-12-2020 Depreciation expense (W-1) 165.82


0.5 marks Accumulated depreciation - Cruise ship 165.82

31-12-2020 Revaluation surplus 165.82–147.20 (W-2) 18.62


2 marks Retained earnings 18.62

31-12-2020 Accumulated depreciation - Cruise ship 165.82


1 marks Cruise ship 165.82

31-12-2020 Revaluation surplus 149.64–18.62 131.02


2 marks Impairment loss (Bal. fig) 103.16
Cruise ship 1,234.18–1,000 234.18

W-1: Rs. in million


Date Description Engine Ship's Body Dry Total
Docking
1/3/2018 Acquisition cost 840.0 535.0 60.0 1,435
1 marks
31/12/18 Depreciation expense (51.84) (15.0) (9.0) (75.84)
1 marks (840–40) ÷50,000× (535–35) ÷ 25×9÷12 60÷5×9/1
(360×9) 2
31/12/19 Depreciation expense (76.80) (20.0) (12.0) (108.80)
1 marks (840– (535–35) ÷25 60
40)÷50,000 × (480×10) ÷5
31/12/19 Carrying value 711.36 500.0 39.0 1,250.36
2 marks
Revaluation surplus
(Bal.) 149.64
31/12/19 Revalued amount 796.49 559.84 43.67 1,400.00
1 marks
31/12/20 Depreciation expense (129.81) (22.57) (13.44) (165.82)
1 marks (796.38–40)÷(50,000–3240 (559.84–35)÷(25–1.75) 43.67÷(5–
–4,800)×(600×12) 1.75)
31/12/20 Carrying value 666.68 537.27 30.23 1,234.18
1 marks
W-2: Depreciation on cost for 2020
Engine (840–40)÷50,000×(600×12) 115.20

Body (535–35)÷25 20.00


Dry docking 60÷5 12.00
147.20
Answer 2
(i) The accounting treatment adopted by accountant for not recording land is incorrect. Allotment of land by
Government is a transfer of a non-monetary asset and should be considered as a government grant. Such non-
monetary grant may be recorded at fair value or at a nominal value. As per NL’s policy, fair value of the land should
be assessed and reported in the financial statements under the head property, plant and equipment (PPE). The
grant was made subject to construction of factory so the resulting deferred income should be recognized in income
on a systematic basis over the useful life of the factory building.

The factory building should also be transferred from capital work in progress to PPE account as the building
is available for use on 1 October 2020. Further depreciation should also be charged from same date i.e. 1
October 2020.

Up to 01 mark for each valid discussion point 3 marks

(ii) The accounting treatment adopted by accountant to record complete building under PPE head is incorrect.
Two floors which have been leased/rented separately so should be accounted for as investment property.
While ground floor used by marketing department should be recorded as property, plant and equipment
under IAS 16 and depreciated over its useful life.

As per NL policy, investment property should be recorded at fair value and changes in fair value should
be taken to statement of profit or loss. Any depreciation already charged on these floors should be
reversed.

Up to 01 mark for each valid discussion point 3 m a r k s

(iii) The accounting treatment adopted by accountant to expense out borrowing cost is incorrect as some
borrowing cost is eligible for capitalization. Power generation plant falls under definition of qualifying
asset as its construction involves substantial period.

Construction of the power plant is financed through specific borrowing so actual borrowing cost incurred
less temporary investment income on the borrowings would be capitalized. However, the borrowing cost
will be capitalized from the date when construction actually started i.e. 1 February 2020 rather than 1
January 2020. Further, the capitalization of borrowing costs should be suspended and charged to the
statement of profit or loss during the three months when work was suspended.

In the statement of profit or loss, borrowing cost on loan and interest income earned from saving account
should be presently separately.

Up to 01 mark for each valid discussion point 4 marks


Answer 3:

Rectifying Original Wrong


Entry for (i)
Advance for Mach. 450,000 Advance for Mach. 450,000 Plant & Mac. 450,000
Plant & Machinery 450,000 Cash 450,000 Cash 450,000
Entry for (ii)
Plant & Machinery 300,000 Plant & Machinery 3,000,000 Plant & Mac. 2,700,000
Repair Expense 300,000 Cash 3,000,000 Repair Exp. 300,000
(2,700,000+300,000) Cash 3,000,000

Entry for depreciation


Accumulated Dep. 15,000 (complete data not available to pass these entries)
Dep. Exp. (W-1) 15,000

Adjustment for depreciation

(Excess)/
Less
Adj. (i) Excess depreciation charged (450,000 x 10%) (45,000)
Adj.(ii) Less depreciation on machine installed (300,000 x 10%) 30,000
Total excess depreciation to be reversed (15,000)
Answer 4
Continued Discontinued
operations operations (W-1)
Earnings attributable to ordinary shareholders during a period (W-5) 761,000,000 155,000,000
Weighted average number of shares in issue during the period (W-1) 16,646,666 16,646,666

EPS 45.71 9.31


Weighted average number of shares during 2016
Weighted
Number of average
Date shares Factor number
x 5/12 x 16/15 x 120/100
Opening Shares on 1 January 10,000,000 x 110/100 5,866,666
Right shares issued on 31 May 2016 (10,000,000/100 x
40) 4,000,000
x 3/12 x 120/100 x
14,000,000 110/100 4,620,000
Bonus shares issued on 31 August 2016 (14,000,000/100
x20) 2,800,000
16,800,000 x 4/12 x 110/100 6,160,000
16,646,666

(W-2) Right issue bonus Fraction (Issued on 31 May)


= Actual cum rights price
Theoretical ex rights price
= 32/30
= 16/15
Theoretical ex-rights price Rs.
Before (100 x Rs.320) 3,200
New (40 x Rs. 25) 1,000
After (140 x Rs.30 (bal)) 4,200

(W-3)
Bonus issue
Before 100
New 20
After 120

(W-4)
Bonus issue
Before 100
New 10
After 110
(W-5) Earnings attributable to ordinary shareholders during a period
Rs. in million
Profit from continuing operations after tax 765
Less: Preference dividends on irredeemable shares (4 x 10 x 10%) (4)
Profit net of Preference dividends 761
Answer 5
Cash flow from Operating Activities Rs. in Million
Profit before tax 1 MARKS (W-1) 13.2
Adjustments for:
Depreciation 2 MARKS (9 + 6.25) 15.25
Financial Charges (interest expenses) 2 MARKS 2.5
Loss on Car 1 MARKS 0.35
Operating cash flow before working capital changes 18.10
Adjustment for working capital changes
(increase)/decrease in current assets
Account Receivables - net 2 MARKS [0-(18-0.9)] (17.10)
Inventory
(10.00)
(increase)/decrease in current liabilities
Account Payable (Creditor) 2 MARKS 14.00
Working capital changes (13.1)

Cash generated from Operations 18.2


Less: Finance charges paid 1 MARKS (W-8) (1.3)
Income Tax paid 1 MARKS (W-7) (5.00)
Drawing (W-5) (2.45)
Net cash flow from operating activities (A) 9.45
Cash flow from Investing Activities
Purchase of Property, Plant and Equipment 2 MARKS (W-2) (103.65)
Proceed from insurance 1 MARKS (W-3) 1.2
Net cash from Investing Activities (B) (102.45)
Cash flow from Financial Activities
Cash contributed by the Partner 50
Proceed from long term loan 25
Net cash flow from financing activities (C) 75
Net increase in cash and cash equivalent 2 (A+B+C) (18.00)
MARKS
0
Add: Cash and cash equivalent at year start
Cash and cash equivalent at end of year 1 MARKS (W-6) (18.00)
WORKINGS
(W-1) Profit before tax Rs. in Million
Sales 136.00
Less: Cost of sales (83.50)
Operating & Selling expenses (37.30)
Add: Miscellaneous Income 0.50
Less: Finance charges (2.50)
Profit before tax 13.2

(W-2)
Dr. Property, Plant & Equipment Account Cr.
Capital (building) 25 Disposal 1.8
Capital 1.4
Cash & Bank 103.65 c/d 128.25
(bal.)
(W-3)
Dr. Disposal Account Cr.
Property, plant and equipment 1.8 Accumulated Depreciation 0.25
P/L 0.35
Cash (bal.) 1.2
(W-4)
Dr. Accumulated Depreciation Account Cr.
Disposal 0.25 b/d 0
c/d 15 Depreciation Expense (9 + 6.25) 15.25
(W-5)
Dr. Partners’ Capital Cr.
Drawings (bal.) 2.45 Cash 50
Building 25
c/d 73.95 Car 1.4
(W-6)
Cash and cash equivalent at end of year
Cash and Bank 5.00
Bank Overdraft (23.00)
(18.00)

(W-7)
Dr. Taxation Expense Cr.
Cash (bal.) 5.00 P/L 6.00
cl. payable 1.00
(W-8)
Dr. Interest Expense Cr.
Cash (bal.) 1.30 P/L 2.50
cl. payable 1.20

Aan
Answer 6
1 mark for each Mcqz

(i) (b) Cash flows on enhancing the asset’s performance

(ii) (d) Recoverable amount


(iii) (a) Increase in central bank discount rates
(iv) (b) Fulfilment value
(v) (c)
(vi) (b) Accrual Accounting
(vii)
(a) Rs. 325,000
(viii) (b) Operating activities

Answer 7
Cocomo Limited
Statement of changes in equity
For the year ended 31 December 2020
Share Share Retained Revaluation
capital premium earnings surplus
--------------- Rs. in million ---------------
Balance as at 1 January 2020 (As given) 250 138 142 45
(250×20%)
Final bonus dividend @ 20% for 50 (50)
2019 2 marks
Interim cash dividend @ 12%for (250+50) ×12%)
2020 1 mark (36)

Total comprehensive income for the year:


Profit for the year 2 marks 45–7 (W-1) 38
Other comprehensive income (10+15) (W-1) (25)
2 marks
Transfer of incremental depreciation 1 marks 5 (5)
Balance as at 31 December 2020 300 138 99 15

W-1: Revaluation of building Building Land


Rs. in million
Revalued amount 158 65
Carrying value (240–60) 180 75
Revaluation loss (22) (10)
Available surplus (20–5) 15 25
Revaluation loss taken to profit or loss 2 marks 7 -
Answer- 8 Rs. Rs.

(i) Profit from soft drinks


Revenue 1 marks 205,000

Less: COS

Op. stock 0.5 marks 40,000

Purchases 0.5 marks 106,500

Wages 37,500

Less: Cl. Stock 1 marks (30,000) (154,000)

Profit 51,000

(ii)

Mayfair Sports and Social Club


Income and Expenditure account
for the year ended December 31, 2003
Incomes Rs.
Profit from soft drinks 51,000
Sale of tickets for annual dinner 120,000
Profit of raffle (9,000 – 3,000) 6,000
Subscription (W-2) 2 marks 112,000
289,000
Expenses
Affiliation fees 5,000
Catering 72,000
Hire of band 15,000
Rent of hall (W-5) 2 marks 72,500
Printing and postage 10,000
Electricity (W-6) 1 marks 30,800
Hon. Secretary expenses 6,100
Repairs to equipment 15,000

Bad debt (W-2.1) 1 marks 7,000


Depreciation (W-4) 1 marks 25,000
(258,400)
Surplus/ (deficit) 30,600
Answer 9

1. Historical
All of the information used in ratio analysis is derived from actual historical results. This
does not mean that the same results will carry forward into the future. However, you can
use ratio analysis on pro forma information and compare it to historical results for
consistency.

2. Inflationary effect
If the rate of inflation has changed in any of the periods under review, this can mean that the
numbers are not comparable across periods. For example, if the inflation rate was 100% in
one year, sales would appear to have doubled over the preceding year, when in fact sales did
not change at all.

3. Accounting policies and estimates


Different companies in a similar industry may have different policies for recording the same
accounting transaction. This means that comparing the ratio results of different companies
may be like comparing apples and oranges. For example, one company might use reducing
balance method while another company uses straight-line depreciation.

4. Business conditions You need to place ratio analysis in the context of the general business
environment. For example, 60 days of sales outstanding for receivables might be considered
poor in a period of rapidly growing sales, but might be excellent during an economic
contraction when customers are in severe financial condition and unable to pay their bills.

5. Interpretation It can be quite difficult to ascertain the reason for the results of a ratio. For
example, an acid test ratio of 2:1 might appear to be excellent, until you realize that the
company just sold a large amount of its stock to bolster its cash position. A more detailed
analysis might reveal that the acid test ratio will only temporarily be at that level, and will
probably decline in the near future.

6. Company strategy It can be difficult to interpret a ratio analysis comparison between two
companies that are pursuing different strategies. For example, one company may be
following a low-cost strategy, and so is willing to accept a lower gross margin in exchange for
more market share. Conversely, a company in the same industry is focusing on a high
customer service strategy where its prices are higher and gross margins are higher, but it will
never attain the revenue levels of the first company.
MOCK EXAMS CAF AUTUMN 2022
Tax Practices
Date: 20th August 2022
3 Hours - 100 Marks
Additional reading time 15 minutes
QUESTION PAPER
Question 1:
Afridi, a resident individual, is working as researcher at Khawaja Institute (KI) which is a non-profit
research institution and is duly recognized by Higher Education Commission. KI is entirely owned
and funded by Burger Limited (BL), a company listed on the Pakistan Stock Exchange.
Details of his monthly remuneration during the year ended 30 June 20X2 are given below:
Rupees
Basic salary 100,000
Medical allowance 10,000
Fair market rent of accommodation 50,000
In addition to the above, he was also provided the following:
Health insurance for Afridi and his dependents as per the terms of employment. For this purpose, KI
is paying annual insurance premium of Rs. 20,000.
Provident fund contribution of Rs. 7,500 per month to a recognized provident fund.
An equal amount was also contributed by Afridi to the fund.

Additional information
(i) On 1 July 2021, Afridi was granted an option to acquire 10,000 shares in BL at a price of Rs. 52.5
per share under an employee share scheme. Afridi bought the option on the same date by paying Rs.
87,500 to KI when the fair market value of the option was Rs. 100,000. He exercised the option on
30 September 2021 when the fair market value was Rs. 65 per Share.
As per the scheme, he was not allowed to sell or transfer the shares before 31 December 2021.
On 31 December 2021, the fair market value of BL’s shares was Rs. 71. On 30 May 20X2, he
sold 5,000 of these shares at Rs. 67.5 per share.
(ii) On 1 July 2021, Afridi obtained an interest free loan of Rs. 750,000 from KI in exchange
for which he agreed to waive the interest receivable on his provident fund balance
maintained with KI. Interest provided on provident fund balance for the year was 8%. The
prescribed benchmark rate is 10%.
(iii) On 31 August 2021, he received leave encashment of Rs. 50,000 relating to previous year.
(iv) During the year, tax of Rs. 80,000 was deducted at source by KI.

Other information relevant to tax year 20X2:


i. On 1 March 20X2, he sold a residential plot situated in Faisalabad for Rs. 9,000,000. The
plot was inherited from his father in 2016. Fair market value of the plot at the time of
inheritance was Rs. 3,500,000.
ii. Afridi received income of USD 2,130 on 25 June 20X2 as under the head income from
other source, he received into his $ account from sale of these goods. Withholding income
tax @ 8% was deducted from the receipt as per the income tax laws of the foreign
country.
iii. Relevant exchange rates were on 25 June 20X2 were Rs.168/$
iv. On 1 June 20X2, Afridi paid Rs. 1,250,000 as donation to a non-profit organization listed
in the 13th schedule of the Income Tax Ordinance, 2001.

Required: Under the provisions of the Income Tax Ordinance, 2001 and Rules made thereunder,
compute the taxable income and net tax payable by or refundable to Afridi for the year ended 30 June
20X2. Show all relevant exemptions, exclusions and disallowances. (18)
Question 2:
Shubina is dissatisfied with the order issued by the Comm. (Appeals) and wants to file an
appeal to the Appellate Tribunal because payment of this amount will cause hardship to her.
Required:
Under the provisions of the Income Tax Ordinance, 2001
I. state the time period within which an appeal may be filed by Shubina Appellate Tribunal. (01)
II. Discuss different types of orders that the Appellate Tribunal may make for disposing of an appeal.
(02)
III. explain what action(s) the Appellate Tribunal may take for ensuring that no undue
hardship will be caused to Shubina because of the payment of this demand. (03)
IV. discuss the option(s) available to Shubina for defending her case, if the Appellate Tribunal
issues an order confirming the amended assessment order issued by the Comm. (Appeal) (02)
V. Describe the formation of Appellate Tribunal. (03)
(b) What is Special Audit Panel? Briefly discuss its provisions under Income Tax Ordinance (05)

Question 3:
(a) Under the Constitution of Pakistan, briefly describe formation of National Finance
Commission. Who may be the member(s) of such Commission? Also state the duties of National
Finance Commission. (05)
(b) XY & Company chartered accountants are calculating tax for the purpose of preparing accounting
entries of their audit client. Identify the threats (if any), In case of identification of threat also discuss
how such threats Can be addressed? (01+02)

Question 4:
Mublas & Co. is an association of persons (AOP) with three members, Fikra, Shikra
and Chikra. Sharing profit and loss in the ratio of 1:1:2 respectively.
During the year, AOP earned profit before tax of Rs. 17,000,000 from its principal business i.e.
trading of garments. In addition, AOP is also involved in purchase and sale of following securities
listed on the Pakistan stock exchange:

Name of Details of purchase Details of sale


investee
company Date No. of Price/shares Date No. of Price/shar es
shares (Rs.) shares (Rs.)
KOX limited 1 Oct 20X6 200,000 400 29 June 20Y0 200,000 450
(Note A)
BBP limited 18 Aug 20X7 55,000 290 20 Dec 20X9 100,000 360
10 Jan 20X8 100,000 300
OIO limited 15 Feb 20Y0 150,000 172 15 March 150,000 156
(Note B) 20Y0

Note A: sales proceed from disposal of these shares was credited to AOP’s bank account on 2
July 20Y0. Note B: Due to shortage o found for making this purchase, AOP borrowed Rs.
10,000,000 in cash from Bhikra, whose business is of lending money at 15 % per annum.
Required:

Under the provisions of income tax ordinance 2001, Compute taxable income & tax liability of AOP
for Tax Year 20Y0.
(08)
Question 5:
James Bond owns an industrial undertaking under the name and style of James & Company (JC) which is
engaged in the business of manufacturing pharmaceutical products. Following information is available for
the year ended 31 December 20X1:
Rs. in '000
Turnover 162,425
Cost of goods sold (108,599)
Gross profit 53,827
Administrative and distribution expenses (44,490)
Marketing expenses (9,883)
Other income 1,780
Profit before tax 1,234
Additional information:

(i) Cost of goods sold includes:


• raw materials of Rs. 3,900,000. No withholding tax was deducted at the time of payment.
• accounting depreciation of Rs. 1,050,000 on plant and machinery.
• provision for slow moving inventory of Rs. 900,000.
(ii) Administrative and distribution expenses include:
• Rs. 338,250 paid to a local hotel for holding annual Eid-Milan party for the employees and their
families.
• Rs. 617,500 paid as penalty to a customer in settlement of his claim for damages under a
contract for the supply of a batch of vaccines. Laboratory tests and in-house investigations
revealed that the level of impurities in the vaccines exceeded the acceptable level as agreed in
the contract.
• Rs. 1,150,000 paid as donation to a hospital established by the local government.
(iii) Marketing expenses include a reward of Rs 250,000. The reward was paid in cash to one of the
salesmen for exceeding his sales target.
(iv) Other income includes:
• dividend of Rs. 87,000. This amount was received from a listed company after deduction of
income tax at the rate of 15% and Zakat of Rs. 15,000 deducted under the Zakat and Usher
Ordinance, 1980.
• gain of Rs. 330,000 on sale of shares in Akash (Pvt) Limited (APL) in November 20X1. 60%
of the shares in APL are owned by the Federal Government. AC purchased these shares in
June 20X0.
Other information:
(i) A second-hand plant was imported from France at a cost of Rs. 1,250,000. Withholding tax of
Rs. 75,000 was deducted at import stage. The plant was installed in the month of September
20X1. AC incurred Rs. 187,500 on the installation of plant which is included in administrative
and distribution expenses.
(ii) Pre-commencement expenditures of Rs. 1,700,000 were charged to accounting profit and loss for
the year ended 31 December 20X0. However, for tax purposes, it has to be amortized over the
period of five years.
(iii) Tax depreciation other than imported plant amounted to Rs. 950,000.
(iv) Income tax deducted by the customers u/s 153 and advance income tax paid u/s 147 during
the year amounted to Rs. 700,000 and Rs. 100,000 respectively.
Required:
Under the provisions of the Income Tax Ordinance, 2001 and Rules made thereunder, compute
total income, taxable income and net income tax payable by or refundable to JC for the tax year
20X2 (19)
Question 6:
(a) In the light of the provisions of Sales Tax Act, 1990 and Rules made thereunder, briefly
explain as to the chargeability/adjustment of sales tax in respect of each of the following
independent matters:

(i) Free provision of taxable goods to the company’s CEO as per the terms of his employment.
(ii) Free replacement of defective parts in the case of taxable goods, sold under warranty.
(iii) Payment of machine fuel by one of the directors using his own credit card. The machine
is used to manufacture taxable goods.
(iv) Taxable goods sold on instalment to a customer at a price inclusive of mark up.
(v) Advance payment received against taxable goods to be supplied to a registered person in next month.
(vi) Local supplies of goods manufactured by a cottage industry.
(vii)Material purchased for the construction of office building.
(viii) Electronic cash register purchased for retail outlet. (08)

(b) Discuss the term “cottage industry” and taxability of supplies of cottage industry. (02)

(c) Under what circumstances the person making the taxable supply shall be jointly and severely
liable with the person receiving taxable goods? (03)

Question 7:

Hammad Sarwar Associates (HSA) is registered under the sale Tax Act, 1990 HSA, is engaged in the
business of manufacturing and supplying of various consumer goods. Following information is
available from HSA's records for the month or August 20X2:

Supplies
Taxable good to registered person 4,325,000
Taxable good to unregistered person 780,000
Exempt good to unregistered person 870,000
Export of taxable good to UK 650,000
Export of exempt good to UK 950000
Purchases
Taxable good from registered person 2,480,000
Taxable good from unregistered person 700,000
Exempt good from unregistered person 260,000
Additional information
(i) taxable goods from registered persons include:
• materials worth Rs. 148,000 which were exclusively used for manufacturing exempt supplies
• materials worth Rs, 337,500. which were exclusively used for manufacturing export related goods.
• goods worth Rs. 75,000 which were purchased in cash from a supplier.
• 500kg of tea purchased at a cost of Rs 180,000 in one kg packing covered under Third
Schedule. Retail price of tea per kg is RS 450 By the end of august 20X2 300 kg were supplies
to an unregister wholesaler at the price of 395 per kg.

(i) Taxable good supplied to unregistered include goods worth Rs. 160,000 which were sold to a
customer who did not provide CNIC or NTN detail these good were Purchased From a registered
supplier for RS 137,500 during August 20X2.
(ii) Following fixed assets were purchased during the month of august 20X2;

Fixed assets Purchased cost Usage


Machine Alpha 1,000,000 To ensure quality
standard of packing for
export
Machine Beta 1,500,000 To manufacture
taxable (local) as well
as exempt (local) good
Office Equipment 500,000 To use in office premises

(iii) Electricity bill of Its 479,725 was paid in cash The bill was inclusive of sale tax of Rs. 77,125.
(iv) Sales tax credit brought forward from last month amounted to Rs 568,790.
(v) Input tax of Rs. 93,000 pertaining to purchase made on 1 February 20X2 was
inadvertently remain unclaimed,
All the above figures are exclusive of sales tax. except where it is specified, Sales payable at the
rate of 17%.

Required:
ln the light of the provisions of Sale Tax Act. 1990 and Rules made thereunder, compute the amount
or sales tax payable by or refundable to HSA and input tax to be carried forward, if any, for the tax
period august 20X2
(18)
Tax Rates

Division1. For individual non-salaried


case/AOP
Taxable Income Rate of Tax
1. Up to Rs. 400,000 0%
2. Rs. 400,000- Rs. 600,000 5% of the exceeding Rs. 400,000
3. Rs. 600,001 - Rs. 1200,000 Rs. 10,000 + 10% of the amount exceeding Rs. 600,000
4. Rs. 1200,001 - Rs. 2400,000 Rs. 70,000 + 15% of the amount exceeding Rs
1,200,000
5. Rs. 2400,001 - Rs. 3,000,000 Rs. 250,000 + 20% of the amount exceeding, 2,400,000
6. Rs. 3,000,000 - Rs. 4,000,000 Rs. 370,000 + 25% of the amount exceeding 3,000,000
7. Rs. 4,000,001 - Rs. 6,000,000 Rs. 620,000 + 30% of the amount exceeding Rs 4m
8. Over Rs. 6,000,000 Rs. 1,220,000 + 35% of the amount exceeding Rs 6m

Division 2 For individual salaried case


M Taxable Income Rate of Tax
1. Up to Rs. 600,000 0%
2. Rs. Rs. 600,001- Rs. 1,200,000 5% of the exceeding Rs. 600,000
3. Rs. Rs. 1,200,001- Rs. 1800,000 Rs. 30,000 + 10% of the amount exceeding Rs. 1,200,000
4. Rs. Rs. 1800,001- Rs. 2500,000 Rs 90,000 + 15% of the amount exceeding Rs 1,800,000
5. Rs. 2,500,001 - Rs. 3,500,000 Rs 195,000 + 17.5% of the amount exceeding, 2,500,000
6. Rs. Rs. 3,500,001- Rs. 5,000,000 Rs 370,000 + 20% of the amount exceeding 3,500,000
7. Rs. 5,000,001 - Rs. 8,000,000 Rs 670,000 + 22.5% of the amount exceeding Rs 5m
8. Rs. 8,000,001 - Rs. 12,000,000 Rs 1,345,000 + 25% of the amount exceeding Rs 8m
9 Rs. 12,000,001 - Rs. 30,000,000 Rs 2,345,000 + 27.5% of the amount exceeding Rs 12m
10 Rs. 30,000,001 - Rs. 50,000,000 Rs 7,295,000 + 30% of the amount exceeding Rs 30m
11 Rs. 50,000,001 - Rs. 75,000,000 Rs 13,295,000 + 32.5% of the amount exceeding Rs 50m
12 Exceeding Rs. 75,000,000 Rs 21,420,000 + 35% of the amount exceeding Rs 75m

Initial allowance 25%


Depreciation rates
P&M 15%
Computer 30%
Motor Vehicle 15%
Building 10%

Tax on Capital Gain under Sec 37 A

12.5% Tax on capital Gain of immovable

property
Where the gain does not exceed Rs 5 m 3.5%
Where the gain exceeds 5 m but does not exceed Rs 10m 5%
Where the gain exceeds 10 m but does not exceed Rs 15m
7.5
% Where the gain exceeds Rs 15m 15%
Mock Solution T
CAF-2 (Tax Practices) Maximum Marks: 100

Question 1 Afridi
Computation of total income, taxable income and net tax payable/refundable
For tax year 2020
Rs.
Income from salary
Basic salary [100,000×12] 1,200,000
Medical allowance [10,000×12] 120,000
Rent free accommodation 600,000
Insurance premium - Exempt -
Employer's contribution to provident fund (W-1) -
Employee share scheme [(71×10,000)–(52.5×10,000)–87,500] 97,500
Concessional loan -
Leave encashment [chargeable on receipt basis] 50,000
Total income from salary 2,067,500

Income from business - foreign source income


Online teaching (2,130×168) 357,840
Add: Withholding tax [2,130×(8/92)×168] 31,117
Total income from business 388,957

Capital gain
Gain on sale of inherited property [(9,000,000–3,500,000)×3/4] 4,125,000
4,125,000

Capital gain from sale of securities


Loss on disposal of shares (W-2) -

Total income 6,581,457


Less: Separate block of income - Capital gain from sale of immovable property (4,125,000)
Taxable income 2,456,457
Less: Donation to non-profit organization covered under 13th Sch.
[Lower of 1,250,000 or 30% of taxable income i.e. Rs. 736,937] (736,937)
Net taxable income 1,719,520

Since salary income is more than 75% of the taxable income, the slab applicable to
salaried individuals shall be applied.

Tax liability
On Rs. 1,200,000 30,000
On remaining Rs. 519,520 @ 10% 51,952
Tax on gain on sale of immoveable property [4,125,000×3.5%] 144,375
Total tax liability 226,327
Less: Reduction in tax liability because of full time professor (W-3) (32,531)
193,796
Less: Foreign tax credit (12,897)
(W-4)
Less: Withholding tax (80,000)
Total tax payable 100,896

W-1
Employer's contribution 90,000
[7,500×12]
In excess of
lower of: (120,000)
- 1/10th salary 120,000
- 150,000 150,000
Taxable under income from salary -

W-2
Cost of 5,000 shares of ZL:
Consideration paid for shares [52.5×5,000] 262,500
Consideration paid to acquire option [87,500×(5,000/10,000] 43,750
Taxable amount included in salary income [97,500×(5,000/10,000)] 48,750
355,000
Consideration received for disposal 337,500
Loss on disposal (17,500)

W-3
Tax payable on salary Rs 2,067,500:
On Rs. 1,800,000 90,000
On remaining Rs. 267,500 40,125
130,125
Tax reduction because of employment as full time professor @ 25% 32,531
W-4
Foreign tax credit lower of;
- foreign tax paid 31,167
- Pakistan tax payable 12,897 12,897
[388,957 ×{193,796/(1,719,520 +4,125,000)}]

Question 2:
(a)
I. Shubina can file to the Appellate Tribunal within 60 days of service of order of the Comm. (Appeals).
II. Where the appeal relates to decision to an assessment order, Appellate Tribunal may make an order to:
a. Affirm, modify or annul the assessment order; or
b. Place the case to the Comm. Or Comm. (Appeals) for further enquiry.
Where appeals relates to decision other than assessment, Tribunal may make an order to affirm, vary or annul the
decision.The Tribunal can increase an assessment or decrease refund after opportunity of showing cause is
provided to taxpayer.
III. The taxpayer has filed an appeal, the tax as per assessment order will still be Payable. However the taxpayer can
apply to the Tribunal for stay of recovery. On receiving application, if Tribunal considers that the recovery of tax
(which is also upheld by the Comm. (Appeals)), shall cause undue hardship to taxpayer, the Tribunal may stay
recovery of tax up to 180 days and will cease to have effect on the expiration of 180 days. It will be done after
providing Comm. an opportunity of being heard. In computing the aforesaid period of 180 days, the period, if any,
for which recovery of tax was stayed by a High Court shall be excluded
IV. Shubina may refer an application with the statement of case to High Court within 90 days of the receipt of
Appellate Tribunal order. The application shall be on a prescribed form and shall state question of law. An
application shall be accompanied by a fee of Rs. 100..
V. The Appellate Tribunal shall consist of a chairperson and judicial and accountant members appointed by the Prime
Minister.
a. Person may be appointed as a judicial member if he
i. Has exercised the powers of a District Judge and is qualified to be a Judge of a High Court;
ii. Is or has been an advocate of a High Court and is qualified to be a Judge of the High Court;
b. Person may be appointed as an accountant member if he is:
i. OIR equivalent to the rank of Chief Comm.
ii. Comm. or Comm. (Appeals) having at least 3 years’ experience.
iii. CA/CMA if he has practiced professionally for at least 10 years.
(b)
• Board may appoint as many special audit panels as may be required to conduct an audit, including a forensic audit
of the income tax affairs of any person and class of person & the scope of such audit shall be as determined by the
Board or the Comm.
• The panel shall comprise of any two or more members from an OIR, firm of CA or CMA Any other person as
directed by the Board.
• The Panel headed by Chairman who shall be an officer of IR.;
• Powers for the purpose of conducting an audit shall only be exercised by an OIR who is member of the panel &
authorized by the Comm.
• Where a person fails to produce any accounts, documents and records, required to be maintained for purpose of
audit or determination of income and tax due thereon, the Comm. may proceed to make best judgment
assessment & Assessment already made or treated to have been made on the basis of return or revised return filed
by the taxpayer shall be of no legal effect.
• If any member of the panel other than Chairman, is absent from conducting an audit, the audit conducted by the
special audit panel shall not be Federal Board of Revenue has the following powers under the law to monitor,
assess, levy and invalid or be called into question merely on account of such absence;
• Functions performed by the OIR as members of special audit panel to conduct audit, shall be treated to be
performed by special audit panel
• Board may prescribe the mode and manner of constitution, procedure and working of the special audit panel

Question 3:(a)

• The President shall constitute an NFC consisting of the Minister of Finance of the Federal Govt., the Ministers of Finance of
the Provincial Govts, and such other persons as may be appointed by the President after consultation with the Governors
of the Provinces.
• It shall be the duty of the NFC to make recommendations to the President as to:
- Distribution between the Federation and the Provinces of the net proceeds of the taxes
- Making of grants-in-aid by the Federal Govt. to the Provincial Govts;
- Exercise by the Federal Govt. and the Provincial Govts of the borrowing powers conferred by the Constitution;
- Any other matter relating to finance referred to the Commission by the President.
(b) Threats
All Audit Clients preparing calculations of current and deferred tax liabilities (or assets) for an audit client for the purpose of
preparing accounting entries that will be subsequently audited by the firm creates a self-review threat.
Actions to address the threat
Audit Clients that are not Public Interest Entities Examples of actions that might be safeguards to address such a self-review
threat when the audit client is not a public interest entity include:
• Using professionals who are not audit team members to perform the service.
• Having an appropriate reviewer who was not involved in providing the service review the audit work or service
performed.
Audit Clients that are Public Interest Entities A firm shall not prepare tax calculations of current and deferred tax liabilities
(or assets) for an audit client that is a public interest entity for the purpose of preparing accounting entries that are material
to the financial statements on which the firm will express an opinion. The examples of actions that might be safeguards in
(to all audit clients) to address self-review threats are also applicable when preparing tax calculations of current and
deferred tax liabilities (or assets) to an audit client that is a public interest entity that are immaterial to the financial
statements on which the firm will express an opinion.
Question 4:
MUBLAS & CO
AOP RESIDENT
FOR THE TAX YEAR 2020
Income from Business 17,000
Capital Gain (w-1) 14,150
Income from other source 10,000
Total income 41,150
Less: Capital gain (SBI) (14,150)
Taxable income 27,000
Tax Up to 6,000,000 1,220
Tax on 21,000,000 x 35% 7,350 8,570
Tax on Separate Block of income (14,150*12.5%) 1768
Total Tax Liability 10,338
W-1) Capital Gain:
KOK Limited (450-400) x 200,000 10,000,000
BBP (360-290) x 55,000 3,850,000
(360-300) x 45,000 2,700,000
OIO (156-172) x 150,000 (2,400,000)
Net Gain 14,150,000
Question 5:
Mr. James Bond
Computation of total income, taxable income, tax payable
Tax period 20X2

Capital gain (W-3) 330,000


Total income 3,734,266
Capital gain in securities (330,000)
Zakat (15,000)
Taxable income 3,389,266
Tax liability NTR (370,000 + {389,266 x 25%}) 467,316
Donation (W-4) (140,195)
Tax on dividend (W-2) 18,000
Tax on capital gain (SBI)W-3 41,250
Advance tax (700,000 +100,000) (800,000)
Withholding tax on input (75,000)
Tax deducted on dividend (FTR) (18,000)
TAX REFUNDALE (506,629)
WORKINGS
W-1 income from business
Profit before tax 1,234,000
ADD: tax not deducted on raw material 780,000
(3,900,000*20%)
A/C depreciation 1,050,000
Provision for bad debt 900,000
Donation 1,150,000
Renewal paid in cash 250,000
Installment of plant 187,500
Less: dividend (87,000)
Capital gain (330,000)
Initial allowance (359,375)
(1,437,500*25%)
Depreciation (80,859)
(1,437,500-359,375)*15%*50%
Amortization of pre-commencement (340,000)
(1,700,000*20%)
Tax depreciation (950,000)
Income from business 3,404,266
W-2 DIVIDENED
Received 87,000
Dividend income 120,000
(87,000+15,000)/0.85
Zakat 15,000
Tax on dividend 18,000
W-3 Capital gain
Capital gain on securities 41,250
(330,000*12.5%)
W-4
Donation 140,195
467,316/3,389,266 x (1,016,780 W-4.1)
W-4.1
Lower of
• 1,150,000
• 1,016,780 (30%*3,389,266)
Notes
(N-1) Expenditure on EID MILAD-on-NABI on employees and their families is allowed as deduction.
(N-2) Penalty on violation is not allowed as deduction, however it is allowed in case it is not due to the violation of any law.

Question 6:

(a)

(i) As this is covered under the definition of supply, this is subject to sales tax by applying tax % on open market price of
these goods.

(ii) The free replacement of defective parts is considered as original supply and not a separate supply. Therefore, such
replacement is not chargeable to tax.

(iii) Payment must be verifiable from the business bank accounts of both the buyer and the seller. In given matter, company
will not be able to obtain input tax on its payment due to non-verifiability of the said payment from the business bank
account of the company.

(iv) Value of supply in this case should be open market price so value of supply does not include the mark up.

(v) Time of supply is the earlier of delivery of goods or the time when payment is received so the advance received is subject
to sales tax in this month.

(vi) These are exempt from sales tax.

(vii) Input tax paid on purchase of construction material for office building is not admissible.

(viii) Input tax paid on purchase of electronic cash register is admissible and registered person shall be entitled to deduct
this input tax from output tax.

(b) Cottage industry Cottage industry means a manufacturer concern, which fulfils each of following conditions namely:
• Does not have an industrial gas or electricity connection
• Is located in a residential area
• Does not have a total labor force of more 10 workers; and
• Annual turnover from all supplies does not exceed Rs 3 million
Local supplies of goods made by cottage industry are exempt from sales tax (6th schedule).
(c) Where a Regd. person receiving a taxable supply from another Regd. person is in the knowledge or has reasonable
grounds to suspect that some or all of the tax payable in respect of the goods supplied would go unpaid such person as well
as the person making the taxable supply shall be jointly and severally liable for payment of such unpaid amount of tax. The
burden to prove this shall be on the tax department.
The Board may exempt any transaction or transactions from the provisions of this section.
Question 7:
Hamaad Sarwar Associates
Computation of Sales Tax Payable/Refundable
For the tax period August 2022
Output tax (W-1) 870,655

Input tax:

Lower of:

b/f 568,790

For the period (W-4) 324,307 893,097; OR

90% of output tax 783,590 (783,590) (870,655


× 90%)

Input tax on PPE (W-4) (217,973)

Sales tax refundable (130,907)

Further tax payable to FBR (780,000 - 118,500 ) ×3% 19,845

Export refund (101,316 + 170,000 + 337,500 × 17%) (328,691) c/f input


tax (893,097-783,590 ) 109,507

W-1) OUTPUT TAX


Supplies to registered person 4,325,000

Supplies to unregistered person 780,000

3rd schedule (450-395) × 300 16,500

Total taxable supplies 5,121,500

Sales tax @ 17% 870,655

W-2) COMMON INPUT TAX


Purchase from registered suppliers 2,480,000

Material for exempt supplies (148,000)

Material for export (337,500)

Purchase in cash (75,000)

500 Kg of Tea [450×500(225,000 - 180,000)] 45,000

Sales for a unregistered person without NTN & CNIC (137,500)

Total purchase 1,827,000


Sales tax @ 17% 310,590

Input tax related to purchase made in Feb 93,000

Sales tax on electricity bill 77,125

Common input tax 480,715


W-3) INPUT TAX ON PPE
Machine Alpha for export 1,000, 000

Sales tax @ 17% 170,000

Machine Beta for taxable & exempt supplies 1,500,000

Sales tax @ 17% 255,000

Note: Input tax cannot be claimed on office equipment

W-4) APPORTIONMENT

Adjusted value of supply Common Input tax Input tax on PPE

Taxable 5,121,500
*324,308 **217,973

Exempt 55,090
870,000
37,027

5,991,500

Zero rated 1,600,000 101,317 -

Total 7,591,500 480,715 255,000

* 5,121,500 × 480,715 **5,121,500 × 255,000

7,591,500 5,991,500
MOCK EXAMS CAF AUTUMN 2022
Cost and Management Accounting
Date: 18th August 2022
3 Hours - 100 Marks
Additional reading time 15 minutes
QUESTION PAPER
SECTION A
Question 1:
Hyundai Limited (HL) manufactures a single product, Theta. During January 2022 it produced 7,290 Kgs of the
Theta. Further relevant information for the month of January 2022 is as follows:

Materials Kgs Price per Kg (Rs.) Rupees

Alpha 3,362 11.00 36,982

Beta 3,290 7.10 23,359

Gamma 1,745 17.00 29,665

8,397 90,006

Loss (1,107)
Yield 7,290

Standard costs of 63 Kgs:

Materials Kgs Price per Kg (Rs.) Rupees

Alpha 30 10.00 300

Beta 24 8.00 192


Gamma 16 15.00 240
70 732
Standard Loss (7)
Standard Yield 63

Required:
Calculate the following variances for each & total material:
(i) Price
(ii) Usage
(iii) Mix (10)
(iv) Yield
Question 2:

ABC Limited deals in manufacturing of tables and chairs. The profit and loss account of the company for
the year ended 30 June 2014 is as follows:

Rs. In ‘000’
Sales 243,000
Cost of goods sold (211,500)
Gross profit 31,500
Operating expenses (57,300)
Net loss before taxation (25,800)
Taxation -
Net loss after taxation (25,800)
Additional information
(i) Selling price per table and chair is Rs. 22,000 and Rs. 8,000 per unit having contribution
margin of 35% and 30% respectively. Tables and chairs are sold in the ratio of 1:4.
(ii) 80% of the operating expenses are fixed while the remaining expenses are directly
attributable the number of units sold.
(iii)The corporate tax is applicable @ 34% while no tax is required to be paid in case of loss.
(iv) The share capital of the company is Rs. 180 million.
Required:
Determine the breakeven sales revenue. (08)
Question 3
CJD Ltd manufactures plastic components for the car industry. The following budgeted information is
available for three of their key plastic components:
Rupees per unit
W X Z
Selling price 200 183 175
Direct material 50 40 35
Direct labour 30 35 30
Units produced and sold 10,000 15,000 18,000
The total number of activities for each of the three products for the period is as follows:

Number of purchase requisitions 1200 1800 2000


Number of set ups 240 260 300
Overhead costs have been analysed as follows:

Receiving / inspecting quality assurance Rs. 1 400 000


Production scheduling/machine set up Rs. 1 200 000
Calculate the budgeted profit per unit for each of the three products using activity based budgeting.
(6)
Question 4:
An engineering company has launched a new product. The production is carried out in batches of 250 units
and the first batch took 800 labour hours. Production manager has estimated that there will be 90% learning
curve on labour hours that will continue till 128 batches after which the learning curve effect will cease.
Cost of production consists of material cost of Rs. 2.400 per unit, labour wages of Rs. 230 per hour and
applied overheads of Rs. 450 per labour hour based on annual capacity of 1 million hours.
Fixed cost of production is estimated at Rs.180 million per annum.
According to the projections of the marketing department, demand would be for 15,000 and 20,000 units
respectively in the first two quarters at selling price of Rs. 3,500 per unit.
Note: The learning index for a 90% learning curve is -0.152
Required:
Prepare the profit and loss account for the first two quarters. (10)

Question 5:
C Ltd is a manufacturing company. In one of the production departments in its main factory a machine hour
rate is used for absorbing production overheads. This is established as a predetermined rate, based on normal
activity. The rate that will be used for the period which is just commencing is Rs 15.00 per machine hour.
Overhead expenditure anticipated, at a range of activity levels, is as follows:

Activity Level Rs.


(Machine Hours)
1500 25 650
1650 26 325
2000 27 900

Required: Calculate:
(i) the variable overhead rate per machine hour;
(ii) the total budgeted fixed overhead;
(iii) the normal activity level of the department; and
(iv) the extent of over-/under-absorption if actual machine hours are 1700 and
expenditure is as budgeted. (10)
Question 6:

Z Limited manufactures a single product, the budgeted selling price and variable cost details of which are as
follows: Rs.

Selling price 15.00


Variable costs per unit:
Direct materials 3.50
Direct labor 4.00
Variable overhead 2.00
Budgeted fixed overhead costs are £60 000 per annum, charged at a constant rate each month. Budgeted
production is 30 000 units per annum.
Required:
In a month when actual production was 2400 units and exceeded sales by 180 units the profit calculate
profit under absorption costing. (4 Marks)
SECTION B
Question 7
Glory Limited (GL) Manufacture two Joint Products MP1 & MP2 from single process. MPI is sold at the
time of completion of process of production department & MP2 is further processed in Refining department
and becomes PK-07. By product (AA1) is also produced in production department. Sale proceeds of AA1 is
treated as reduction in cost of main product. Joint costs are allocated on the basis of net realizable values.
Normal Loss is 3 % of input in both departments and are occurred at start of Process.
Scrap value of normal loss is Rs. 12 per kg.

Production Department Refining Department


Material @ 210/kg 18,900,000 -
Direct Labor @ 420 Rs per 4,578,000 2,275,340
hour
Production overheads 2,356,500 993,450
Output:
MP1(kgs) 42,000
AA1(kgs) 2,000
MP2(kgs) 40,500
PK-07 (kgs) 37,000
Units Sold:
MP1(kgs) 41,200
PK-07 (kgs) 35,000
Opening WIP (kgs) - -
Closing WIP (kgs) 80 % - 1,300
Sale Price per kg:
MP1 1,400
PK-07 1,790
AA1 45

Packing cost of Rs. 100 for MP-1 and Rs. 70 for PK-07 will be incurred.
Required:
Determine Product wise Profit/(loss) for MP1 and PK-07. (16)
Question 8

Sophisticated packing Limited (SPL) has received an order for supply of 2 million packing wrappers.
The rapping material is in the form of a film. The manufacturing involves two processes. Printing
and lamination. One packing wrapper requires 0.04 running meter of film in each process.
Printing:
Ink and a chemical are applied to the first layer of film. One running meter of film consumes 2 grams
of ink and 4 grams of chemical. 1 kg of film has 35 running meters. Normal wastage during the
process of printing is estimated at 3%. In addition, it is estimated that approximately 1,200 meters
of film would be wasted at the timing of setting up the machine.
Lamination:
During the process of lamination, a second layer of the film is applied on the first layer using glue.
1 kg of glue is used to laminate 250 meters. There is normal wastage of 2% during lamination.

The raw material prices are as follows:

Raw material Price per kg


(Rs.)
Printing and lamination film 260
Ink 180
Chemical 150
Glue 110

Both processes would require 1,000 productive labour hours in total (at 100% efficiency) comprising
of 25% skilled and 75% unskilled workers who are paid @ Rs. 35,000 and Rs 15,000 per month
respectively. Both skilled and unskilled workers work an average of 200 hours per month at 90%
and 85% efficiency respectively.
Printing and lamination overheads are estimated at Rs.5 and Rs. 3 per running meter respectively.

Required:
Compute the selling price of the order if SPL wishes to earn a 20% margin on sale price
(20)
Question 9
Owais Limited (OL) deals in a single product. For 2013-14, the sale was projected at 40,000 units. OL
followed a policy of maintaining safety stock of 500 units and placed orders on the basis of Economic Order
Quantity (EOQ). The total carrying costs amounted to Rs. 900,000 whereas carrying cost per unit per month
was Rs. 30.
The projections for 2014-15 are as follows:
• Annual sales would be 52,800 units.
• Expected sale price of the product is Rs. 4,000 per unit and expected contribution margin is
5% of sales.
• Carrying cost per unit is expected to increase by 20%.
• Fixed costs of the ordering department would increase by 15% over last year whereas
variable costs associated with processing an order would increase by 10%.
• There would be no change in the policy for maintenance of safety stock.
Required:
(a) Compute the following for the year 2014-15:
(i) Economic Order Quantity
(ii) Total ordering and carrying costs
(11)
(b) Suggest revised safety stock level based on the assumption that lead time usage and the related
probabilities are as follows:

(5)
Market condition Units Probability
Favorable 5,000 0.15
Moderate 4.500 0.50
Slow 4,000 0.35
Section A
Answer 1

Page 1 of 9
Answer 2

Page 2 of 9
Answer 3

Page 3 of 9
Answer 4

Answer 5

Page 4 of 9
Answer 6

Page 5 of 9
Section B
Answer 7

Page 6 of 9
Page 7 of 9
Answer 8

Order quantity 2,000,000


Finished running meters (0.04 ×2,000,000) 80,000
Lamination film input in meters (80,000÷98%) 81,633
Printing film input in meters (81,633÷ 97%) 84,158
Set-up consumption 1,200
85,358

Meters Kg Rate Amount in


Rs.
Raw materials
Printing 85,385 *2,439 260 634,140
Lamination film 81,633 *2,332 260 606,320
Ink (2 x 84,158 ÷ 1,000) 168 180 30,240
Chemical (4 x 84,158 ÷ 1000) 337 150 50,550
Glue (81,633 ÷ 250) 327 110 35,970
1,357,220
Labour cost:

Skilled labour [250÷ (200×90%) ×35,000] 48,611


Unskilled labour [750÷ (200×85%) ×15,000] 66,176
114,788
Overheads
Printing Overheads (85,358×5) 426,790
Lamination Overhead (81,633×3) 244,890 671,689

Total cost 2,143,697


Profit margin (25% on cost) 535,924
Selling price 2,679,621
*85,358÷35=2,439 **81,633÷35=2,332

Page 8 of 9
Answer 9

Page 9 of 9
Rise School of Accountancy
MOCK EXAMS CAF AUTUMN 2022
Business Law
Date: 16th August 2022
3 Hours - 100 Marks
Additional reading time 15 minutes
QUESTION PAPER
Instructions to examinees:
(i) Answer all questions.
(ii) Answer in black pen only.
(iii) Multiple Choice Questions must be answered in answer script only.

Question 1:
Select the most appropriate answer from the options available for each of the following Multiple
Choice Questions (MCQs). Each MCQ carries ONE mark.

(i) The President if deems necessary to take immediate action, he has power to make an Ordinance when
the National Assembly is not in session. The Ordinance shall stand repealed after
a) One hundred and Thirty days
b) One hundred and Twenty days
c) One hundred and Sixty days
d) One hundred and eighty days

(ii) Part-II of the Constitution of the Islamic Republic of Pakistan is about


a) Amendment of Constitution
b) Relations between Federation and Provinces
c) Fundamental Rights and Principles of Policy
d) Islamic Provisions

(iii) Khalid resident of Bahawalpur sends a letter by post to Abid of Lahore offering to sell his Horse for
Rs. 200,000/-. The letter is posted on 10 February 2021 and reached Abid on 17th February 2021.
Abid wrote his acceptance on 18th February. Abid sends his acceptance by post on 20th February
2021, Khalid receives this letter of acceptance on 25th February. When is communication of
Acceptance completed as against Khalid?
a) The communication of acceptance is complete on 17th February 2021
b) The communication of acceptance is complete on 18th February 2021
c) The communication of acceptance is complete on 20th February 2021
d) The communication of acceptance is complete on 25th February 2021

(iv) Riaz agrees to sell his 10 marla house located in Bahria Town to Asif for Rs. 500/-. Is it a valid
contract?
a) It is voidable contract as consideration is not adequate
b) No, this is not enforceable by law
c) It’s a void agreement
d) It is a valid contract

(v) Moeed, an infant, borrows Rs. 50,000 from Mahad which afterwards could not be covered by Mahad
because Moeed was not competent to contract. Moeed on attaining age of majority ratify his
agreement with Mahad. Whether this ratification is valid and Mahad can recover from Moeed?
a) Such ratification is not valid. For a minor’s agreement, being void ab initio cannot be made
valid by subsequent ratification
b) Minor can ratify any agreement entered into during age of minority after attaining age of majority
c) Subsequent ratification by any person is valid no matter that person is minor or major
d) None of the above
Rise School of Accountancy

(vi) Shareef who has badly in need of money offered to sell his House worth Rs 5 million to Badmaash for
Rs. 500,000. Before possession of house delivered to Badmaash, Shareef received an offer of Rs. 5.5 million
and refused to carry out the contract on the ground of inadequacy of consideration. Is Shareef liable to
Badmaash for damages?
a) Shareef is not liable because such agreements are voidable
b) Consideration was not adequate, so agreement is void
c) Consideration should be at least one thirds of original value of house, so Sharee is not liable for any
damages
d) Shareef is liable to pay for damages, because agreement to which the consent of the party is freely
given is not void merely because the consideration is inadequate
(vii) In which of the following relationships there is NO presumption that a person is in a position to
dominate the will of another person
a) Husband and wife
b) Doctor and Patient
c) Fiancé and Fiancée
d) Employer and employee

(viii) Rana Sb buys a parrot from Khan sb, which both believe to be a talking parrot. But afterwards they
revealed that this parrot is not a talking parrot. Is the contract valid?
a) The contract is absolutely valid and enforceable
b) The contract is not void because there was unilateral mistake
c) The contract is absolutely void, because there is a mutual mistake of both the parties as to the quality
of the subject-matter
d) The contract is valid because there was free consent of both parties

(ix) Faqeer Khan obtains two loans of Rs. 40,000 each from Ameer Khan, one of which is guaranteed by
Ghareeb Khan.
Faqeer Khan sends to Ameer Khan a sum of Rs. 40,000 without any direction regarding appropriation
towards the loans. Ameer Khan appropriates Rs. 40,000 towards the loan not guaranteed by Ghareeb Khan
and Rs 10,000. Ghareeb Khan objects to such appropriation. Is such objection by Ghareeb is valid?
a) Ghareeb’s objection is valid because debt guaranteed by Ghareeb should be settled first and then other
b) Ghareeb’s objection is valid, because in the absence of any express or implied instructions from the
debtor regarding appropriation, the surety is entitled to apply the payment
c) Ghareeb’s objection is not valid, because in the absence of any express or implied instructions from
the debtor regarding appropriation, the creditor is entitled to apply the payment
d) Ghareeb’s objection is valid, because in the absence of any express or implied instructions from the
debtor regarding appropriation, surety is entitled to apply the payment

(x) Imran Khan borrowed Rs. 300,000 from Altaf and Shahbaz jointly and promised to repay the amount on
February 20, 2021. Who can claim performance if Altaf died before the due date?
a) Contract is discharged and no need to perform
b) only Shahbaz can claim performance from Imran Khan
c) Legal representatives of both promisees jointly can claim performance
d) Shahbaz and legal representatives of Altaf jointly can claim performance
Rise School of Accountancy
(xi) Which of the following is a deceptive marketing practice?
a) Refusing to deal while having a dominant position
b) Predatory pricing driving competitors out of a market, prevent new entry, and monopolize the market
c) Fraudulent use of another's trademark, firm name, or product labelling or packaging
d) Boycotting or excluding any other undertaking from the production, distribution or sale of any goods
or the provision of any service
(xii) An individual committed the offence of money laundering, he shall be punished with rigorous imprisonment
for a term which shall not be less than
a) two years but may extend up to ten years
b) one year but may extend up to ten years
c) one year but may extend up to fifteen years
d) one year but may extend up to twenty years

(xiii) A company (legal person) committed the offence of money laundering and one of its directors was found
guilty. The company shall be liable to fine which may extend up to
a) Rs. 30 million
b) Rs. 70 million
c) Rs. 100 million
d) Rs. 50 million

(xiv) While having lunch at a restaurant, Adil found some gold coins under his chair which he took into his custody.
Latif, the restaurant owner, demanded Adil to hand over the gold coins to him.
Which of the following statements is correct?
a) Adil should hand over the gold coins to Latif as he would be regarded as a bailee being the restaurant
owner
b) Adil should not hand over the gold coins to Latif as Adil is the pledgee to the owner of the gold coins
c) Adil can keep the gold coins as he has become the bailee to the owner of the gold coins
d) Adil can keep the gold coins as he has become the agent to the owner of the gold coins

(xv) Farah draws a bill of exchange on Asad which states “Asad, pay Rs. 100,000 to Sana’s successors or order”.
The bill of exchange drawn by Farah shall be considered to be:
a) invalid because it does not mention the date by which payment must be made
b) invalid because the parties indicated in the bill of exchange are not certain
c) valid because it contains an order to pay
d) valid since all the essential elements of a negotiable instrument are satisfied
Rise School of Accountancy
Question 2:
Explain the type of contracts in the following agreements under the Contract Act, 1872:
(i) A coolie in uniform picks up the luggage of A to be carried out of the railway station without
being asked by A and A allows him to do so.
(ii) Obligation of finder of lost goods to return them to the true owner.
(iii) A contract with B (owner of the factory) for the supply of 10 tons of sugar, but before the supply is
affected, the fire caught in the factory and everything was destroyed. (06)

Question 3:
What will be rights with the promisor in following cases? Explain with reasons:
(a) Mr. X promised to bring back Mr. Y to life again.
(b) A agreed to sell 50 kgs of apple to B. The loaded truck left for delivery on 15th March but
due to riots in between reached A on 19th March.
(c) An artist promised to paint on the fixed date for a fixed amount of remuneration but met
with an accident and lost his both hands.
(d) Abhishek entered into contract of import of toys from China. But due to disturbance
in the relation of both the countries, the imports from China were banned (04)

Question 4:
Mr. Moeed was introduced to the benefits of partnership of M/s ABC & Co. with the consent of
all partners. After attaining majority, more than six months elapsed and he failed to give a public
notice as to whether he elected to become or not to become a partner in the firm. Later on, Mr.
Luqman, a supplier of material to M/s ABC & Co., filed a suit against M/s ABC & Co. for recovery
of the debt due.
In the light of the Partnership Act, 1932, explain:
a) To what extent Mr. Moeed will be liable if he failed to give public notice after attaining
majority?
b) Can Mr. Luqman recover his debt from Moeed? (05)

Question 5:
a) Mr. Zaid and Mr. Yameen are partners in a partnership firm. Mr. Zaid introduced Majid (an
employee) as his partner to Zahid. Majid remained silent. Zahid, a trader believing Majid as
partner supplied 50 Laptops to the firm on credit. After expiry of credit period, Zahid did not
get amount of Laptop sold to the partnership firm. Zahid filed a suit against Zaid and Majid
for the recovery of price. Does Majid is liable for such purpose? (03)
b) Mr. Luqman while drafting partnership deed taken care of few important points. Luqman wants
to know the list of information which must be part of partnership deed drafted by him. What
list of information to be included in partnership deed? (03)
Rise School of Accountancy
Question 6:
a) Under the Payment Systems and Electronic Fund Transfers Act, 2007 state the circumstances
under which the State Bank of Pakistan may revoke the designation of a payment system as
designated payment system. (06)
b) CFE Limited (CL) entered into a contract with PRC Limited (PL) and agreed that in case of
any dispute, both parties to the contract will resolve the matter through arbitration. In the light
of Arbitration Act, 1940 advise CFE and PRC regarding the provisions which are deemed to
be included in an arbitration agreement, in absence of any contrary intention expressly
provided in the arbitration agreement itself. (08)

Question 7:
a) Discuss with reasons, whether the following persons can be called as a ‘holder’ under the
Negotiable Instruments Act, 1881:
(i) X who obtains a cheque drawn by Y by way of gift.
(ii) A, the payee of the cheque, who is prohibited by a court order from receiving the amount
of the cheque.
(iii) M, who finds a cheque payable to bearer, on the road and retains it.
(iv) B, the agent of C, is entrusted with an instrument without endorsement by C, who is the payee.
(v) B, who steals a blank cheque of A and forges A’s signature (05)
b) Dawood owes Rs. 100,000 towards Sharafat and undertakes to endorse a bill of exchange in
his favour in order to settle the debt. Under the provisions of the Negotiable Instruments Act,
1881 explain to Sharafat the essentials of a valid endorsement as he wants to ensure that the
bill of exchange is properly endorsed. (04)
c) Mansoor bought readymade garments worth Rs. 500,000 from Saqlain on credit today 8
March 2022. The amount is payable after three months. Mansoor wants to issue a negotiable
instrument in satisfaction of his debt to Saqlain without involving a third party for the payment.
Under the provisions of the Negotiable Instruments Act, 1881 identify the type of negotiable
instrument which Mansoor may issue to Saqlain in satisfaction of his debt. Also prepare a draft
of the said instrument. (04)
(You may assume necessary details for the preparation of the negotiable instrument)

Question 8:
a) An agreement without consideration is void with certain exceptions. State the exceptions. (04)
b) Sabir bought Akram’s car factory in Lahore on Akram’s representation that thirty thousand
cars are assembled at his factory annually. Sabir later found that the factory has a capacity to
manufacture twenty thousand cars only per annum. Sabir now wants to rescind the contract on
the ground that his consent was obtained by misrepresentation
Under the provisions of Companies Act, 1872 list any two circumstances in which a party whose
consent was obtained by misrepresentation cannot rescind the contract. (02)
Rise School of Accountancy
Question 9:
a) Basmati Rice Traders (BRT) agreed to supply best quality basmati rice to Dawood Pakwan
Centre (DPC) on 8 March 2023 for the marriage ceremony of Faysal. However, on due date,
BRT failed to fulfil their obligation. Under the provisions of the Contract Act, 1872 explain
the rights available to DPC under the above situation. Assume time was the essence of the
contract. (03)
b) Salman rented his shop to Qadeer for a period of one year at an agreed sum of Rs. 50,000 per
month. After the first five months, Qadeer defaulted in making payment of the rent. Burhan, a
known trader, being concerned with the strained relationship between Salman and Qadeer, paid
the rent with good intention. Subsequently, on Qadeer’s refusal to reimburse the amount,
Burhan filed a suit against him on the grounds that he made the payment to Salman which
Qadeer was legally bound to make and being a quasi-contract Buran is entitled to the
reimbursement. Explain whether Burhan is justified in his suit. (03)

Question 10:
a) Distinguish between civil law and criminal law giving two examples of each. (04)

b) What do you understand by delegated legislation? Give two advantages and disadvantages
of such legislation. (03)

Question 11:
On 1 February 2022 Qaiser offered, by a letter, to sell his house to Amanat for Rs. 900,000. Amanat
received the letter on 6 February 2022. On 8 February 2022 Amanat posted the letter of acceptance
to Qaiser. The letter reached Qaiser on 12 February 2022.
Qasier wrote a letter of revocation of his offer and posted it to Amanat on 6 February 2022. The
letter reached Amanat on 10 February 2022.
Required:
Under the provisions of the Contract Act, 1872 briefly describe:
(i) When the communication of the offer and acceptance and the revocation of the offer was
completed as against Qaiser and Amanat under the above circumstances. (04)
(ii) Whether a binding contract was created between Qaiser and Amanat. (02)

Question 12:
a) What is meant by term ‘Critical infrastructure’? (03)
b) State punishments for unauthorized access, copying and damage to critical infrastructure
information system or data. (03)
Question 13:
What are the essential requisites of a valid offer of performance? What is the effect of refusal
by the promisee to accept tender of goods and money, from the promisor? (06)
(THE END)
Rise School of Accountancy
B Law
Mock solution
Answer-01
1. B
2. C
3. C
4. D
5. A
6. D
7. A
8. C
9. C
10. D
11. C
12. B
13. C
14. C
15. B

Answer-02
(i) It is an implied contract and A must pay for the services of the coolie detailed by him. Implied
contracts come into existence by implication. Most often the implication is by law and or by
action.
(ii) Obligation of finder of lost goods to return them to the true owner cannot be said to arise out
of a contract even in its remotest sense, as there is neither offer and acceptance nor consent.
These are said to be quasi-contracts.
(iii) The contract is a void contract.
“A contract which ceases to be enforceable by law becomes void when it ceases to be
enforceable”. Thus, a void contract is one which cannot be enforced by a court of law.

Answer-03
a) The contract is void because of its initial impossibility of performance.
b) Time is essence of this contract. As by the time apples reached B they were already
rotten. The contract is discharged due to destruction of subject matter of contract.
c) Such contract is of personal nature and hence cannot be performed due to occurrence of an
event resulting in impossibility of performance of contract.
d) Such contract is discharged without performance because of subsequent illegality nature of
the contract
Answer-04
a) As per Partnership Act, 1932, at any time within six months of his attaining majority,
or of his obtaining knowledge that he had been admitted to the benefits of partnership,
whichever date is later, such person may give public notice that he has elected to become or
that he has elected not to become a partner in the firm, and such notice shall determine his
position as regards the firm. However, if Moeed fails to give such notice, he shall become a
partner in the firm on the expiry of the said six months.
Moeed becomes personally liable to third parties for all acts of the firm done since he was
admitted to the benefits of partnership.

b) Moeed’s share in the property and the profits of the firm remains the same to which he was
entitled as a minor. In the instant case, since, Moeed has failed to give a public notice, he
shall become a partner in the M/s ABC & Co. and becomes personally liable to Mr. Luqman,
a third party.

Answer-05
a) As per Partnership Act, 1932, where a man holds himself out as a partner, or allows others
to do it, he is then stopped from denying the character he has assumed and upon the faith of
which creditors may be presumed to have acted. A person may himself, by his words or
conduct have induced others to believe that he is a partner or he may have allowed others to
represent him as a partner. In the given case, Majid (the Manager) is also liable for the price
because he becomes a partner by estoppel or holding out as per Partnership Act, 1932.
b) A partnership deed usually sets out the following:
 Firm name
 Place or principal place of business of the firm
 Names of any other places where the firm carries on business
 The date when each partner joined the firm
 Number of partners
 Names in full and permanent addresses of partners
 Duration of partnership (if any)
 Purpose of the partnership
 Rights and duties of the partners.
 Amount of capital that each partner should put into the business, and keep in the
business until the partner retires or the partnership is dissolve
Answer-06
a) The State Bank may revoke the designation of a DPS if it is satisfied that:
(i) the DPS has ceased to operate effectively as a PS;
(ii) the operator of the designated system has knowingly furnished information or documents
to the State Bank in connection with the designation of the PS which is or are false or
misleading any material particular;
(iii) the operator or settlement institution of the DPS is in the course of being wound up or
otherwise dissolved, whether in Pakistan or elsewhere;
(iv) any of the terms and conditions of the designation or requirements of the Act has been
contravened; or
(v) the State Bank considers that it is in the public interest to revoke the designation.
(vi) The State Bank shall not revoke a designation without giving the operator of the DPS an
opportunity to be heard. However, the State Bank may suspend the designation of a PS
without notice pending the final order, if an immediate systemic risk is involved

b) An arbitration agreement, unless a different intention is expressed therein, shall be deemed to


include the following provisions (set out in First Schedule of the Act):
(i) Unless otherwise expressly provided, the reference shall be to a sole arbitrator.
(ii) If the reference is to an even number of arbitrators, the arbitrators shall appoint an
umpire not later than one month from the latest date of their respective appointments.
(iii) The arbitrators shall make their award within four months after entering on the reference
or after having been called upon to act by notice in writing from any party to the
arbitration agreement or within such extended time as the Court may allow.
(iv) If the arbitrators have allowed their time to expire without making an award or have
delivered to any party to the arbitration agreement or to the umpire a notice in writing
stating that they cannot agree, the umpire shall forthwith enter on the reference in lieu of
the arbitrators.
(v) The umpire shall make his award within two months of entering on the reference or
within such extended time as the Court may allow.
(vi) The parties to the reference and all persons claiming under them shall subject to the
provisions of any law for the time being in force, submit to be examined by the
arbitrators or umpire on oath or affirmation in relation to the matters in difference and
shall, subject as aforesaid, produce before the arbitrators or umpire all books, deeds,
papers, accounts writings and documents within their possession or power respectively,
which may be required or called for, and do all other things which, during the
proceedings on the reference, the arbitrators, or umpire may require.
(vii) The award shall be final and binding on the parties and persons claiming under
them respectively.
(viii) The cost of the reference and award shall be in the discretion of the arbitrators or
umpire who may direct to, and by whom, and in what manner, such costs or any part
thereof shall be paid, and may tax or settle the amount of costs to be so paid or any part
thereof and may award costs to be paid as between legal practitioner and client
Answer-07
a) As per section 8 of the Negotiable Instruments Act, 1881 ‘holder’ of a
Negotiable Instrument means any person entitled in his own name to the
possession of it and to receive or recover the amount due thereon from the
parties thereto.
On applying the above provision in the given cases
(i) Yes, X can be termed as a holder because he has a right to possession and
to receive the amount due in his own name.
(ii) No, he is not a ‘holder’ because to be called as a ‘holder’ he must be
entitled not only to the possession of the instrument but also to receive the
amount mentioned therein.
(iii) No, M is not a holder of the Instrument though he is in possession of the
cheque, so is not entitled to the possession of it in his own name.
(iv) No, B is not a holder. While the agent may receive payment of the
amount mentioned in the cheque, yet he cannot be called the holder
thereof because he has no right to sue on the instrument in his own
name.
(v) No, B is not a holder because he is in wrongful possession of the instrument.
a) The following are essentials of valid indorsement:
 It must be on instrument itself, if no space is left on the back of the indorsement,
further indorsements are signed on a slip of paper attached to the instrument called
‘allonge’.
 It must be signed by the endorser for the purpose of negotiation. Signature of the
endorser on the instrument without any additional words is sufficient.
 No particular form of words is necessary for an indorsement
 It must be completed by the delivery of the instrument with the intention of
passing the property in it.
 Negotiation by indorsement must be of the entire instrument. Indorsement for part
of the amount is invalid.
b) Mansoor would issue a promissory note as time instrument to Saqlain in satisfaction of his debt

Answer-08
Exceptions:
i) Natural love and affection
An agreement without consideration shall not be void if:
• it is expressed in writing and registered under the relevant law; and
• it is made on account of natural love and affection between parties standing in a near
relation to each other.
ii) Promise to compensate past voluntary services
An agreement without consideration shall not be void if it is a promise to compensate, wholly or
in part, a person who has already voluntarily done something for the promisor, or something
which the promisor was legally compellable to do
iii) Time barred debt
An agreement without consideration shall not be void if:
• it is a promise, made in writing and signed by the debtor or his agent; and
• it relates to a payment (wholly or in part) of time barred debt. A time barred debt is debt
which a creditor cannot enforce payment of due to law of limitation
iv) Completed gifts
Gift is the transfer of certain existing movable or immovable property made voluntarily and
without consideration by one person (i.e. donor) to another person (i.e. donee) and accepted by or
on behalf of the donee. Any completed gift actually made between donor and donee is valid,
although without consideration. A gift is executed in writing through a deed duly registered

b) Exceptions to rescind the contract under misrepresentation: A party cannot rescind the
contract where:
• Party whose consent was caused had means of discovering truth with ordinary diligence;
• Party gave the consent in ignorance of misrepresentation

Answer-09
a. Time being essence of the contract, following would be the rights of Dawood Pakwan
Centre (DPC) under the circumstances:
▪ Contract would be voidable at the option of Dawood Pakwan Centre (DPC)
▪ Dawood Pakwan Centre (DPC) may insist that Basmati Rice Traders (BRT) should
deliver building material. However, in order to claim compensation on account of the
delayed supply, Dawood Pakwan Centre (DPC) shall have to give notice to Basmati
Rice Traders (BRT) of their intention to do so at the time of acceptance of performance
at any time other than earlier agreed.
▪ Dawood Pakwan Centre (DPC) may decide not to accept performance beyond the
stipulated time and rescind the contract and claim compensation for any damages
which it may have sustained due to nonfulfillment of the contract by Basmati Rice
Traders (BRT).
b. To constitute a quasi-contract and be entitled for reimbursement, following conditions
must be satisfied:
• the person who made the payment must have his own interest in the payment; and
• the other person must be bound by law to pay.
As Burhan does not seem to have any interest in the payment and therefore, So he is not justified
in his suit
Answer-10

Example: Civil laws


• property disputes (Transfer of property Act)
• work-related disputes (employment law)
• accusations of negligence (negligent behaviour) (Tort)
Example: Criminal laws
• Pakistan Penal Code
• Anti-Money Laundering Act
• Prevention of Electronics Crimes Act

b) In Delegated Legislation power is given to an Executive (a minister or public body to make


subordinate or delegated legislation for specified purposes only) e.g. local authorities are
given statutory powers to make bye-laws which apply within a specific locality

Advantages of delegated legislation


Time: Parliament does not have time to examine matters in detail
Expert opinion: Much of the content of delegated legislation is technical and is better worked out in
consultation with professional, commercial or industrial groups outside Parliament.
Flexible: Delegated legislation is more flexible than an Act of Parliament. It is far simpler to
amend a piece of delegated legislation than to amend an Act of Parliament
Disadvantages of delegated legislation
The main criticism of delegated legislation is that it takes law making away from the
democratically elected members. Power to make law is given to unelected civil servants and
experts working under the supervision of a government minister.
Because delegated legislation can be produced in large amounts the volume of such law
making becomes unmanageable and it is impossible to keep up-to-date
Answer-11
i) The communication of offer is complete on 6 February because the letter containing the
offer reaches the offeree on 6 February, 2022.
• The communication of acceptance is complete against the offeror on 8 February because the
letter of acceptance is posted on 8 February, 2022.
• The communication of acceptance is complete as against the acceptor on 12 February
because the letter of acceptance is received by the offeror on 12 February.
ii) Qasier’s revocation is not valid because Qasier only revoke his offer before
communication of acceptance is completed against Qaiser that was completed on 8
February.

Answer-12
a) critical infrastructure" means critical elements of infrastructure namely assets, facilities,
systems, networks or processes the loss or compromise of which could result in:
▪ major detrimental impact on the availability, integrity or delivery of essential services
including those services, whose integrity, if compromised, could result in significant loss
of life or casualties, taking into account significant economic or social impacts; or
▪ significant impact on national security, national defence, or the functioning of the state.
The Government may also designate any private or Government infrastructure in accordance with the above
objectives, as critical infrastructure.
Answer-13
Every offer/tender of valid performance must fulfill the following conditions:
(i) It must be unconditional
(ii) It must be made:
• At a proper time.
• At a proper place.
• Under such circumstances that the person to whom it is made may have a reasonable
opportunity of ascertaining that the person by whom it is made is able and willing there and
then to do the whole of what he is bound by his promise to do.
(iii)If the offer is an offer to deliver anything to the promisee, the promisee must have a
reasonable opportunity of ascertaining that the thing being offered is the one which the
promisor is bound to deliver. Effect of refusal to accept offer of performance: Where a
promisor has made an offer of performance to the promisee, and the offer has not been
accepted, the promisor is not responsible for non-performance, nor does he thereby lose his
rights under the contract.
MOCK EXAMS CAF AUTUMN 2022
Financial Accounting and Reporting II
Date: 15th August 2022
3 Hours - 100 Marks
Additional reading time 15 minutes
QUESTION PAPER
Question 1:

You have recently joined the accounts department of Towellers Limited (TL). Junior accountant has prepared
financial statements for the year ended June 30, 2018. However, he is not much confident about accounting
treatment of following transactions:

a) Research and development project of product A was started on July 1, 2017.Following expenditure were
incurred on this project:
i. Costs incurred up to 30-09-17 when a project review was undertaken and it was assessed that development
of the new product was economically and technically viable = Rs.500,000.
ii. Costs incurred between 01-10-17 to 31-03-18 when product was launched immediately after completion
of project (including Rs.50,000 on staff trainingand Rs.100,000 on promotional advertising) = Rs.750,000.
The new product has an estimated useful life of four years. Currently the total cost of Rs.1.25million has
been capitalized as an intangible asset and no amortization has been charged. (03)
b) Research and development project of product B was started on July 1, 2017 andfollowing costs
were incurred:
• Background investigation work (01-07-17 to 31-10-17): Rs.120,000
• Initial development work (01-11-17 to 31-12-17): Rs.240,000
• Second phase development work (01-01-18 to 31-03-18): Rs.660,000
• Product launch cost (May 2018): Rs.70,000
• Staff training cost (June 2018): Rs.30,000
Product B was assessed commercially viable on January 1, 2018 and product development was completed
by the end of March 2018. Though the product was widely launched in May 2018 butfirst batch was sold in
July 2018. The product has an estimated life of five years. Currently the total cost of Rs.1.12 million has
been capitalized as an intangible asset and no amortization has been charged so far. (03)
c) During the year sales staff of TL, as a result of its extensive efforts, developed a customer list. A
competitor of TL has offered Rs.1.5 million for this list. Junior accountant has recognized this list as
intangible asset at Rs.1.5 million being fair market price. (02)
d) During 2018 TL recognized two unique brands as intangible assets. The first brand was acquired for
Rs.400,000 on April 1, 2018 and on the same date an internally generated brand was also recognized. An
expert valued the internally generated brand at Rs. 500,000 on April 1, 2018 and this value was recognized
in revaluation surplus. Both brands are estimated to have four-year useful life. No amortization for the year
has been charged so far. (02)
e) During the year TL incurred Rs.3 million on nationwide advertisement. The advertising consultant believes
that such aggressive advertising campaign would result in increased sales for at least next four years. Junior
accountant has recognized it as intangible asset and charged amortization of Rs.0.75 million [Rs.3m / 4] (02)
Required: Discuss in accordance with relevant IFRS about whether the treatments followed by junior
accountant in respect of above transactions are correct or not?

Question 2:

On January 1, 2018, Alpha Limited (AL) bought 4,000 TFCs for Rs. 120 each. Transaction taxes paid on
this transaction were Rs. 5 each. Interest at 12% of face value (i.e. Rs. 100 each) is received at end of every
year. The effective rate of interest is 8%. The fair values of TFCs were as follows:

Date Fair value(Rs.)

December 31, 2018 485,000


December 31, 2019 490,000
On January 1, 2020 AL sold the TFCs for Rs. 490,500

Required:
Journalize all above transactions over all relevant years if:

a. AL's business model is to hold TFCs till the redemption date.


b. AL's business model is to hold TFCs until redemption but also to sell them if investments with higher
returns become available.
c. AL's business model is to trade TFCs in the short term. (14)

Question 3:
Following are the statements of profit or loss for the year ending December 31, 2018:
Baap Ltd. Beta Ltd.
------ Rs.’000-----
Sales 190,000 150,000
Cost of sales (110,000) (96,000)
Gross profit 80,000 54,000
Distribution cost (11,000) (9,600)
Admin expenses (12,000) (6,000)
Finance cost (4,000) (5,600)
Other income 6,200 1,700
Profit before tax 59,200 34,500
Tax (21,500) (12,600)
Profit after tax 37,700 21,900

Following additional information is available:


(i) Baap Ltd. acquired 80% shares of Beta Ltd. on June 1, 2018 for:
Cash payment of Rs.40 million (including Rs.3.8 million transaction costs)
Share exchange of 2 shares of Baap Ltd. for every 5 shares of Beta Ltd. acquired
At acquisition date share prices of Baap Ltd. and Beta Ltd were Rs.30 and Rs. 25respectively.
Non-Controlling interest is to be valued at fair value.
(ii) Baap Ltd. also acquired 40% shares of Bhateeja Ltd. on October 1, 2018. Bhateeja Ltd’s profit after tax for the
year amounts to Rs. 7 million. Impairment loss on investment in associate at December 31, 2018 is estimated at
Rs. 0.3 million.
(iii) At acquisition date fair value of an internally generated brand of Beta Ltd. was Rs. 6million. Reliable
estimate of its useful life was made at 5 years.
(iv) On July 1, 2018 Baap Ltd. gave a loan of Rs. 40 million to Beta Ltd. at an annual interest of10%. Interest is paid
semiannually. Both companies have properly accounted for this interest.
(v) Beta Ltd. had been selling goods to Baap Ltd. for some years at a markup of 30%. Since acquisition, markup
was reduced to 25%. Average sales were Rs. 2 million per month. Out of the post-acquisition sales, some goods
having sales value of Rs. 1 million were stillheld in Baap Ltd.’s stock at year end.

(vi) Impairment loss of goodwill for the year was Rs. 500,000.
(vii) Beta Ltd.’s other income includes Rs. 200,000 which specifically relates to January 2018.
(viii) All incomes and expenses, except discussed in points (iv) and (vii), are assumed to occurevenly throughout the
year.
(ix) Balances extracted from balance sheet of Beta Ltd. as at December 31, 2017 areas follows:
Rs.’000
Share capital (Rs. 10 each) 30,000
Share premium 3,000
Retained earnings 29,300

Required:
(a) Calculate goodwill on acquisition. (03)
(b) Prepare consolidated statement of profit or loss for the year ending December 31, 2018. (14)

Question 4:
Decent Limited (DL) sells various cheap, but expensive-looking electronic items. All goods are sold with a six-month
warranty, provided by DL. DL’s suppliers are Maldives Limited (ML) and Armenia Limited (AL). AL also offers a 6-
month guarantee on allgoods sold to customers, thus any returns by customers to DL will be passed onto AL for
fulfillment. ML offers no such refunds, although it has a past practice of refunding unsatisfied customers most of the
times. During the year ended December 31, 2015, total sales made by DL, ML and AL were Rs. 0.5 million, Rs. 7 million
and Rs. 5 million respectively. All sales are assumed to be equal for each month and monthly sales occur at end of month.
At year end following estimates are available in respect of refunds.
Most likely now Highest ever Lowest ever
DL 15% 30% 10%
ML 5% 15% 1%
AL 10% 18% 8%
Required:

Discuss how above refunds should be dealt with in the financial statements for the year ending December31, 2015
for each company separately (11)
Question 5:

President Limited (PL), a Pakistan based company, is preparing its financial statements for the year ended 30
June 2019. PL purchased Property, Plant & Equipment in Australia forAUD (Australian Dollar) 7.8 million. 10%
advance payment was made on 1 December 2018and 70% payment was made on 1 February 2019 on transfer of
title and possession of the property. The remaining amount was paid on 1 March 2019. PL uses the revaluation
modelfor subsequent measurement of Property Plant & Equipment and account for revaluation onthe net
replacement method. Depreciation is provided on straight line basis with useful life of 5 years.

On 30th June 2019, an independent value determined that fair value of the property was AUD8
million.

Following spot exchange rates are available:

Date 1-Dec-2018 1-Feb-2019 1-Mar 2019 1-April-2019 30-June-2019

AUD 1 Rs. 100 Rs. 105 Rs. 108 Rs. 110 Rs. 116
Required:

Prepare journal entries in the books of PL for the year ended 30th June2019. (07)

Question 6:

Pasha & Company is a manufacturing company with the following operating segments:

Operating External Internal Net Profit/(loss) Liabilities


Segments Revenue Revenue Assets For the year

----------------------------------------------------Rs in Million
A 70 45 50 100 75
B 300 300 1000 3150 2000
C 453 12 900 2200 400
D 50 90 1700 1350 235

Required:
As per IFRS-8, identify and explain whether the above operating segments are reportable or not? (06)
Question 7:
Suraj Ltd. (SL) signed the lease agreement with Chand Ltd. (CL) for the lease of equipmenton 1-
Sept-2017.SL made the equipment available to CL on 1-Jan-2018.
Following information pertains to the lease agreement:

Lease term 5 years.


Lease rentals Rs.44,991 per annum payable on every 31-December.
Expected useful life of machine 8 years.
Residual value of machine Rs.20,000 at the end of useful life.
Implicit rate 9% per annum.
Market interest rate 10% per annum.

• Documentation charges paid by SL and CL were Rs.15,000 and Rs.5,000 respectively.


• Ownership will be transferred to CL at end the of lease term.
Required:

Prepare the following for the books of CL:


a. Prepare all journal entries for the year ending 31-Dec-2018.
b. Extracts of income statement and balance sheet as at 31-Dec-2018.
Note to financial statements as per IFRS-16 for the year ending 31-Dec-2018 (14)

Question 8:

XYZ is a listed company engaged in the business of manufacturing of leather goods. The applicable rate for
the Co. is 40% in year 1994, 1995 and 1996.

The relevant information for calculation of deferred tax is as under:


1. Charitable donations are recognized as expenses when they incurred and are not deductible for tax
purposes. (1995: Rs. 500, 1996: Rs. 350)
2. In 1995, the entity was notified by the relevant tax authorities that they intend to pursue action against
the entity with respect to sulphur emissions. Although as at December, 1996 the action had not yet
come to court the entity recognized a liability of Rs. 700 in 1995 being its best estimate of the fine
arising from the action. Fines and penalties are not deductible for tax purposes. (1996: Also Rs. 700)
3. In 1992, the entity incurred Rs. 1,250 of costs in relation to the development of new product. These
costs were deducted for tax purposes in 1992. For accounting purposes, the entity capitalized this
expenditure and amortized it on the straight-line basis over five years. At 31st December 1994, the
balance of these product development costs was Rs. 500.
4. In 1995, the entity entered into an agreement with its existing employees to provide health care benefits
to retirees. The entity recognizes as an expense the cost of this plan as employees provide service
(1995: Rs. 2,000, 1996: Rs. 1,000). No payments to retirees were made for such benefits in 1995
and 1996.
Healthcare costs are deductible for tax purposes when payments are made to retirees.
5. Buildings are depreciated at 10% a year on a straight-line basis for tax purposes. Motor vehicles are
depreciated at 25% a year on a straight-line basis for tax purposes. A full year’s depreciation is
charged purposes in the year that an asset is acquired.
6. There was an addition to Buildings of Rs. 6,000 during the year 1995 and in vehicles of Rs. 15,000
during the year 1996.
7. The cost of buildings is Rs. 50,000 and vehicles Rs. 10,000 at the end of 1994. The tax allowance on
the assets already taken is Building Rs. 40,000 and vehicles Rs. 5,000.
8. The accounting profit for 1995 is Rs. 8,775 and for 1996 is Rs. 8,740. The extracts of balance sheets
for 1994, 1995and 1996 are as under:

1994 1995 1996

Assets Carrying Amount Carrying Amount Carrying Amount


Product development costs 500 250 -

Property, plant & equipment 36,000 37,200 44,400

Equity and liabilities


Fine payable - 700 1,400
Liability for healthcare benefits - 2,000 3,000

Required:

a) Calculate opening balance of D.T.L/D.T.A as on January 1, 1995.


b) Calculate current tax expense for the year ended 31 December, 1995 and 1996.
c) Calculate deferred tax expense for the year ended 31 December, 1995 and 1996. (19)
Solution of Mock FAR II
Answer 1:
(i) In accordance with IAS 38, research cost is recognized as expense and developmentcost
is capitalized only after certain capitalization criterion is met. Research cost ofRs. 500,000
incurred up to assessment of project being viable should be charged off as expense
[1 mark]. Development cost incurred after conditions aremet should be capitalized but it
should not include advertisement cost and staff training cost. Therefore, Staff training cost
of Rs. 50,000 and product launching costof Rs. 100,000 should be charged as expense
[1 mark] and Rs. 600,000 [750,000 – 50,000 – 100,000] should be capitalized as intangible
asset. [1 mark] Moreover, amortization of Rs. 37,500 [600,000/4 x 3/12] should be
charged as expense [1 mark] for the year 2018.

(ii) In accordance with IAS 38, research cost is recognized as expense and developmentcost
is capitalized only after certain capitalization criterion is met. Research cost of Rs. 120,000
[0.5 mark] and initial development work of Rs. 240,000 incurred up to assessment of
project being viable should be charged off as expense [0.5 mark]. Development cost
incurred after conditions are met should be
capitalized but it should not include advertisement cost and staff training cost. Therefore,
Staff training cost of Rs. 30,000 and product launching cost of Rs. 70,000 should be charged
as expense [0.5 mark] and Rs. 660,000 should be capitalized as intangible asset [0.5 mark].
Moreover, amortization should commence when asset is available for use [0.5 mark],
therefore, amortization of Rs. 33,000 [660,000/5 x 3/12] should be charged as expense
[0.5 mark] for the year2018.
(iii) Internally generated customer list should not be recognized as intangible asset as
its cost cannot be distinguished from establishing the business as a whole [1 mark].
Therefore, Rs. 1.5 million should not be recognized as intangible asset. [1 mark]

(iv) Purchased brand should be recognized as intangible asset however, internallygenerated


brand should not be recognized as intangible asset its costcannot be distinguished from
establishing the business as a whole [1 mark]. Therefore, purchased brand should be
recognized as intangible asset at Rs. 400,000 [0.5 mark] and internally generated brand
should not be recognized as asset. Moreover, amortization of Rs. 25,000 [400,000/4 x 3/12]
should be charged as expense [0.5 mark] for the year 2018.

(v) Advertisement expenditure should not be recognized as intangible asset as JL cannot


exercise control [1 mark] over its benefits. Rs. 3 million should be charged as expense.
[1 mark]

Solution of Mock Sir jamshed Butt (1.22) 1 of 14 22-Aug-22


Answer 2
(a) Asset shall be measured at amortized cost
Dr. Cr.
-------- Rs. -------
01-01-18 Investment [4,000 x Rs. 125] 500,000
Cash 1 mark 500,000
[Initial recognition]

31-12-18 Investment 40,000


Investment income (W-1) 1 mark 40,000
[Investment income for 2018]

31-12-18 Cash [400,000 x 12%] 48,000


Investment 0.5 mark 48,000

[Interest received for 2018]

31-12-19 Investment 39,360


Investment income (W-1) 1 mark 39,360
[Investment income for 2019]

31-12-19 Cash 48,000


Investment 0.5 mark 48,000
[Interest received for 2019]

01-01-20 Cash 490,500


Profit on disposal 1 mark 7,140
Investment (W-1) 483,360
[Sale of investment]
(b) Asset shall be measured at fair value through OCI

Dr. Cr.
-------- Rs. -------
01-01-18 Investment [4,000 x Rs. 125] 500,000
Cash 0.5 mark 500,000

Solution of Mock Sir jamshed Butt (1.22) 2 of 14 22-Aug-22


[Initial recognition]

31-12-18 Investment 40,000


Investment income (W-1) 0.5 mark 40,000
[Investment income for 2018]

31-12-18 Cash 48,000


Investment 0.5 mark 48,000

[Interest received for 2018]

31-12-18 Fair value reserve [OCI] 7,000


Investment (W-1) 1 mark 7,000
[Fair value loss 2018]

31-12-19 Investment 39,360


Investment income (W-1) 0.5 mark 39,360
[Investment income for 2019]

31-12-19 Cash 48,000


Investment 0.5 mark 48,000

[Interest received for 2019]

31-12-18 Investment (W-1) 13,640

Fair value reserve [OCI] 1 mark 13,640

[Fair value gain 2019]

Solution of Mock Sir jamshed Butt (1.22) 3 of 14 22-Aug-22


01-01-20 Cash 490,500

Investment 1 mark 490,000

Profit on disposal 500

[Sale of investment]

01-01-20 Fair value reserve [OCI] 6,640


Investment (W-1) 0.5 mark 6,640
[Cumulative gain reclassified to P&L]

(c) Asset shall be measured at fair value through P&L


Dr Cr
. .
-------- Rs. -------
01-01-18 Investment [4,000 x Rs. 120] 480,000

P&L(transaction costs) [4,000 x Rs. 5] 1 mark 20,000


Cash 500,000
[Initial recognition]

31-12-18 Investment [485,000 - 480,000] 5,000

P&L 0.5 5,000


mark
[Fair value gain for 2018]

31-12-18 Cash 48,000

Investment income 0.5 48,000


mark
[Interest received for 2018]

31-12-19 Investment [490,000 - 485,000] 5,000

P&L 0.5 5,000


mark
[Fair value gain for 2019]

Solution of Mock Sir jamshed Butt (1.22) 4 of 14 22-Aug-22


31-12-19 Cash 48,000

Investment income 0.5 mark 48,000

[Interest received for 2019]

01-01-20 Cash 490,500

Investment 1 mark 490,000


Profit on disposal 500
[Sale of investment]

W-1

Closing Air Value


Date Opening balance Interest Payment Balance Fair value Reserve OCI

31-12-18 500,000 40,000 48,000 492,000 485,000 (7,000) (7,000)


31-12-19 492,000 39,360 48,000 483,360 490,000 6,640 13,640

Solution of Mock Sir jamshed Butt (1.22) 5 of 14 22-Aug-22


Answer 3
(a)
Goodwill Rs. 000 Rs. 000
investment
Cash [40-3.8] 36,200.00
Shares [3,000*30*2/5*80%] 28,800.00 65,000.00
Fair value of NCI [3000*20%*25] 15,000.00
80,000.00
Less: Net assets acquired
share capital (30,000.00)
share premium (3,000.00)
brand (6,000.00)
Retained earnings [29.3+21.9-11.825] (39,375.00)
(78,375.00)
1,625.00
goodwill 1,625.00
(b)
Group Consolidated SOCI
for the year ending December 31, 2018 Rs.000
Sales [190+(150*7/12)-14] 263,500
cost of sales [110+(96*7/12)-14+.2] (152,200)
GP 111,300
Distribution cost (11+9.6*7/12) (16,600)
Admin expenses (12+6*7/12+.5+.7) (16,700)
Finance cost (4+(5.6-2)*7/12+2-2) (6,100)
Other income (6.2+(1.7-.2)*7/12-2) 5,075
Share of profit from associate(w-1) 400
PBT 77,375
Tax (21.5+12.6*7/12) (28,850
PAT 48,525
Profit attributable to:
Shareholders of Baap Ltd 46,440
NCI(w-2) 2,085
48,525
(w-1)
share of profit from associate
7*3/12*0.4 700
Impairment (300)
400
(W-2)
PAT [(21.9-.2+2)*7/12-2] 11,825
Impairment loss (500)
amortization of brand (6/5*7/12) (700)
URP (1*25/125) (200)
10,425
20% 2,085

Solution of Mock Sir jamshed Butt (1.22) 6 of 14 22-Aug-22


Answer 4

1. Decent Limited (DL


DL offers guarantee to all its customers (1 mark), therefore, there is a present legal obligation to refund
(0.5 mark) as a result of past event i.e. sale (0.5 mark). Also, it is apparent from recent estimate and past
records that outflow of economic resources will take place (0.5 mark). Hence a provision shall be
recognized in books of DL at end of 2015 in respect of refunds. Since guarantee period for sales of 1st half
of 2015 has expired, therefore, provision shall be recognized for sales in 2nd half of the year as follows:

[500,000 x 6/12 x 15%] = Rs. 37,500 (1 mark)


Furthermore, a reimbursement is available in respect of goods purchased from AL. Therefore, a
reimbursement asset should be recognized in respect of above refunds only if recovery from AL is
virtually certain (0.5 mark). In this case, the amount of asset shall not exceed Rs. 37,500 (1 mark).
However, if recovery is probable then contingent asset should be disclosed (0.5 mark).

2. Maldives Limited (ML)


ML does not offer guarantee to all its customers however it has mostly refunded and thus raised a valid
expectation of refund among customers (1 mark), therefore, there is a present constructive obligation
to refund (0.5 mark) as a result of past event i.e. sale. (0.5 mark) Also it is apparent from recent estimate
and past records that outflow of economic resources will take place (0.5 mark). Hence a provision shall
be recognized in books of ML at end of 2015 in respect of refunds. Since there is no guarantee period,
therefore, provision should be made for whole year sale as follows:

[7,000,000 x 5%] = Rs. 350,000 (1 marks)

3. Armenia Limited (AL)


AL offers guarantee to all its customers (1 mark), therefore, there is a present legal obligation to refund
(0.5 mark) as a result of past event i.e. sale.(0.5 mark) Also it is apparent from recent estimate and past
records that outflow of economic resources will take place (0.5 mark). Hence a provision shall be
recognized in books of AL at end of 2015 in respect of refunds. Since guarantee period for sales of 1st half
of 2015 has expired, therefore, provision shall be recognized for sales in 2nd half of the year as follows:

[5,000,000 x 6/12 x 10%] = Rs. 250,000 (1 marks)

Solution of Mock Sir jamshed Butt (1.22) 7 of 14 22-Aug-22


Answer 5

President Limited (PL)

Date Accounting Head Debi Credi


t t
12/1/2018 Advance 78,000,000
Bank 78,000,000

2/1/2019 Advance 573,300,000


Bank 573,300,000

2/1/2019 PPE (bal.) 815,100,000


Advance 651,300,000
Payable 163,800,000

3/1/2019 Payable 163,800,000


Exchange loss (P&L) 4,680,000
Bank 168,480,000

6/30/2019 Depreciation 67,925,000


Accumulated Dep 67,925,000

6/30/2019 Accumulated Dep 67,925,000


PPE 67,925,000

6/30/2019 PPE 180,825,000


Revualtion Surplus(OCI) 180,825,000
(M-1 Each (8000000*116)-747,175,000
entry)

Solution of Mock Sir jamshed Butt (1.22) 8 of 14 22-Aug-22


Answer 6
Pasha & Co.
As Per Revenue:
1320
Total Revenue (Internal + External) =
70+45+300+300+453+12+50+90

Segment Revenue Percentage Reportable


A 115 115/1320*100 = 8.71% 
B 600 600/1320*100 = 45.4% 
C 465 465/1320*100 = 35.22% 
D 140 140/1320*100 = 10.61% 
As Per Profit:
Total Profit = (100+3150+2200+1350) 6800
A's percentage of Profit = 100/6800*100 1.47%
As per Assets
Total Assets = (50+75+1000+2000+900+400+1700+235) 6360
A's percentage as per Total Assets = 50/6360*100 0.79%
Conclusion: B, C and D are reportable segments
75% Threshold:
Total External Revenue 873

Reportable Operating Segments External Revenue


B 300
C 453
D 50
Total 803

Percentage = 803/873*100 91.98%

As external revenue is greater than 75%, hence there is no need to add additional operating segments.

Solution of Mock Sir jamshed Butt (1.22) 9 of 14 22-Aug-22


Answer 7 (a)
Chand Limited.
Date Particulars Dr. Cr.

01-Jan-18 Right of use (W-1) 175,000


Lease liability [M-1] 175,000
[Initial recognition of lease]
1-Jan-18 Right of use 5,000
Bank [M-1] 5,000
[Payment of IDC]
Depreciation
31-Dec-18 [(180,000- 20,000
20,000) / 8] 20,000
Accumulated depreciation [M-1]
[Depreciation charge for the year]
31-Dec-18 Lease liability (W-2) 29,241
Finance charge (W-2) 15,750
44,991
Bank
[Payment of rental] [M-1]
W-1 PV of LP [Discount at 9%]
PV of rentals 175,000
PV of excepted payment for GRV -
PV of LP 175,000
[44,991(1-(1+0.09)-5 /0.09] [M-1]
W-2 Lease Schedule
Date Open. Bal. Payment Interest Principal Clos. Bal.
31-Dec-18 175,000 44,991 15,750 29,241 145,759
31-Dec-19 145,759 44,991 13,118 31,873 113,886

(b)
INCOME STATEMENT - Extracts [M-1]
Rs.
Finance Charge 15,750
Depreciation 20,000
BALCANCE SHEET – [M-1.5]
Extracts
Non-Current assets
Right of use [180,000 – 20,000] 160,000

Non-Current liabilities
Lease liability 113,886
Current liabilities
Lease liability 31,873

Solution of Mock Sir jamshed Butt (1.22) 10 of 14 22-Aug-22


(c)

NOTES TO THE ACCOUNTS


5 - Lease
Lease term is 5 years. Residual value of Rs. 75,000 is expected at end of useful life. Implicit
rate is9%. Rs. 44,991 is payable per annum at end of every year. [M-1]

For the year: [M-1.5] Rs.


Depreciation 20,000
Finance charge 15,750
Total cash outflow for leases 49,991

Lease assets: [M-1]


Carrying amount 160,00
0
Addition to right of use 180,00
0

Maturity analysis:
Rs.
Undiscounted lease payments are as follows: [M-2]
1 year 44,991
2 years 44,991
3 years 44,991
4 years 44,991
179,964

Solution of Mock Sir jamshed Butt (1.22) 11 of 14 22-Aug-22


Answer 8

a) Opening balance of deferred tax liability as on 1st January 1995 (W-1) Rs. 8,600. 0.5

b) Calculation of current tax


1996 1995
Profit before tax 8,740 8,775
Add: Accounting depreciation (W-3) 7,800 4,800 0.5
Fines and penalties 700 700 0.5
Charitable donations 350 500 0.5
Healthcare benefits expense 1,000 2,000 0.5
Amortization on development expenditures (1,250/5 years) 250 250 0.5
10,100 8,250
Less: Tax depreciation (W-4) 11,850 8,100 0.5
(11,850) (8,100)
Taxable Profit 6,990 8,925
Current tax @ 40% 2,796 3,570 0.5

c)
1996 1995
Deferred tax expense 1,120 420 0.5

(W-1) Calculation of Deferred tax – as on December 31,1994


Carrying Amount Tax Base T.T.D/ (D.T.D) D.T.L/ (D.T.A)
P.P&E (-):(W-4) 36,000 15,000 21,000 0.5
Development costs 500 - 500 0.5
21,500
Deferred Tax Liability (21,500 X 40%) 8,600 0.5
Calculation of deferred tax - as on December 31,1995

Carrying Tax T.T.D / D.T.T/


Amount Base (D.T.D) (D.T.L
P,P&E (-):(W-4) 37,200 12,900 24,300 0.5
Fine payable (W-8) 700 700 - 0.5
Development costs 250 - 250 0.5
Liability for healthcare benefits 2,000 - (2,000) 0.5
22,550
Deferred tax liability (22,550 x 40%) 9,020 0.5

Solution of Mock Sir jamshed Butt (1.22) 12 of 14 22-Aug-22


Calculation of deferred tax - as on December 31, 1996
Carrying T.T.D/ D.T.L/
amount Tax Base (D.T.D) (D.T.A)
P,P&E (-):(W-4) 44,400 16,050 28,350 0.5
Fine payable (W-8) 1,400 1,400 - 0.5
Development costs - - -
Liability for healthcare benefits 3,000 - (3,000) 0.5
25,350
Deferred tax liability (25,350 x 40%) 10,140 0.5

(W-2)

Dr. Deferred tax liability/asset Cr.


b/d (1/1/95) (W-1) 8,600
c/d (31/12/95) (W-1) 9020 Deferred tax expense (bal.) 420 2
b/d (1/1/96) 9,020
c/d (31/12/96) (W-1) 10,140 Deferred tax expense (bal.) 1,120 2

(W-3)
Dr. Property, Plant & Equipment-BV (accounting rules) Cr.
b/d (1/1/95) 36,000 Dep. (bal.) 4,800
Additions 6,000 c/d (31/12/95) 37,200 1
b/d (1/1/96) 37,200 Dep. (bal.) 7,800
Additions 15,000 c/d (31/12/96) 44,400 1

(W-4)
Dr. Property, Plant & Equipment (as per tax) Cr.
b/d (1/1/95) 15,000 Dep. (W-5) (W-6) (5,600+2,500) 8,100
(50,000-40,000) + (10,000-5,000)
Additions 6,000 c/d (31/12/95) (bal.) 12,900 0.75
b/d (1/1/96) 12,900 Dep. (W-5) (W-6) (5,600+6,250) 11,850
Additions 15,000 c/d (31/12/96) (bal.) 16,050 0.75

(W -5) Calculation of tax depreciation on building 1995


On opening (50,000 x 10%) 5,000
On additions (6,000 x 10%) 600
5,600 0.5
1996
On openings (56,000 x 10%) 5,600 0.25

(W-6) Calculation of tax depreciation 1995

On opening (10,000 x 25%) 2,500 0.25


1996

On openings (10,000 x 25%) 2,500


On additions (15,000 x 25%) 3,750
6,250 0.5

Solution of Mock Sir jamshed Butt (1.22) 13 of 14 22-Aug-22


(W-7)

Dr. Provision (Liability for health care benefits) Cr.


b/d (1/1/95) 0
c/d (31/12/95) 2,000 Expense 2,000
b/d (1/1/96) 2,000
c/d (31/12/96) 3,000 Expense 1,000

(W-8)
Dr. Fine expense Cr.
b/d (1/1/95) 0
c/d (31/12/95) 700 Expense 700
b/d (1/1/96) 700
c/d (31/12/96) 1,400 Expense 700

Solution of Mock Sir jamshed Butt (1.22) 14 of 14 22-Aug-22


MOCK EXAMS CAF AUTUMN 2022
Managerial and Financial Analysis

Date: 17th August 2022


3 Hours - 100 Marks
Additional reading time 15 minutes
QUESTION PAPER

Section A

Q.1 Select the most appropriate answer from the options available for each of the following Multiple -Choice Questions.

(i) Best Cook Limited (BCL) is engaged in processed frozen food (food) production. The market for food is
highly competitive where key players are striving hard to increase their profits and there has been no
major new competitor entered into the market in the past 2 years. The prices are mostly stable and
competitors are trying to attract targeted customers by using different tactics.
BCL’s product is in stage of lifecycle and it is recommended to .

(a) growth, differentiate product


(b) maturity, differentiate product
(c) maturity, market product aggressively
(d) decline, leave the market (02)

(ii) There is significant gas shortage in A-town. Tech Appliances (TA) is considering importing
economical variants of electric ovens and geysers to meet the ever-increasing market demand. TA has contacted
supplier in China who has agreed to supply these products shortly. Currently, no competitor is offering such
variants in A-town.
By using Porter’s five forces model, it can be inferred that:

(a) threat of new entrants is high and bargaining power of buyers is high
(b) threat of new entrants is high and bargaining power of buyers is low
(c) threat of new entrants is low and bargaining power of buyers is high
(d) threat of new entrants is low and bargaining power of buyers is low (01)

(iii) Which of the following statements is correct about aggressive working capital funding policy?

(a) All permanent assets as well as part of the fluctuating current assets are financed by long-
termfunding
(b) All permanent assets as well as part of the fluctuating current assets are financed by short-
termfunding
(c) All fluctuating current assets as well as some of the permanent parts of current assets are
financed by short-term funding
(d) Only fluctuating current assets are financed by short-term funding (1.5)
(iv) Which of the following is NOT the step for implementation of a risk management system?

(a) Demonstrating commitment to risk management and allocating appropriate resources


(b) Developing an appropriate implementation plan including deadlines
(c) Identifying where, when and how different types of decisions are made and by whom
(d) Ensuring that the organization’s arrangements for managing risk are clearly understood and practiced
(01)

(v) Hamza Limited’s production process is very mechanized. The process involves four machines. Machine
Maintenance Department (MMD) informs that if one of the machines breakdowns, it will not affect
significantly on the production process. However, breakdown of more than one machines together would stop
the production process. According to the MMD, the chances of sudden breakdown of anyone of the four
machines is 3%.

The Manager Production wants to sign a contract with a company in order to have backup machines
if any such unforeseen situation arises. Before signing such contract, he wants to know the
probability ofbreaking down of any two machines together?

(a) 0.0009
(b) 0.9409
(c) 0.0216
(d) None of the above (1.5)

(vi) Which of the following statements is correct about the firewall?

(a) It is a device installed at the boundary of a company to prevent unauthorized physical access
(b) It is a device installed at the boundary of an entity’s system to protect it against the unauthorized access
(c) It is a kind of wall built to prevent fires from damaging the corporate assets
(d) None of the above (01)

(vii) Danish Ibrahim is considering a start-up business. He has performed the feasibility of business and is
very optimistic about its future prospects. The business would require the investment of Rs. 5 million
for financing capital assets and working capital. Danish has Rs. 2 million as savings and looking for
Islamic mode of financing for the remaining amount. He does not want any interference from finance
provider in making business decisions.
Danish should opt for:

(a) Murabaha
(b) Ijarah
(c) Mudaraba
(d) None of the above (01)
(viii) Platinum Limited introduced performance-based bonus scheme (bonus scheme) for the sales team last
year. The management expected that bonus scheme would improve the overall productivity level by at
least 10%. It has gathered following data of number of units sold by each sales personnel before and after
the introduction of bonus scheme:

Before bonus scheme After bonus scheme


Sales officer
(units sold) (units sold)
Khadija 500 400
Shahzaib 400 450
Zainab 450 400
Kashan 600 650
Saleem 450 500
Rida 550 500
Salman 400 950

If management takes the mean to determine the average increase in productivity level, then it can be concluded that:

(a) bonus scheme is effective as it results in an average increase in productivity level byapproximately 15%
(b) bonus scheme is not effective as it fails to meet the management expectation of at least 10%increase in
productivity level of sales team
(c) bonus scheme is effective as it results in an average increase in productivity level byapproximately
13%
(d) bonus scheme does not have any effect as increase in number of units sold is compensated by decrease in
number of units sold

(ix) Continuing from above case scenario, if management takes a decision on the basis of mean for average
increase in productivity level, it can be said that:

(a) the management has rightly chosen mean for decision as it is only method that accounts for all values of the data
(b) the management has rightly chosen mean for decision as it is least affected if there are any outliers in the data
(c) the management has wrongly chosen mean for decision as it is more affected if there are any outliers in the data
(d) the decision would remain unaffected if management had chosen any other method for average (1.5)

(x) Assume that two regression coefficients are known. Which of the following formulas will help in
computing the coefficient of determination?
(a) b×d
(b) √b × d
(c) b÷d
(d) None of the above (01)
(xi) Tracking employee training, skills and performance appraisals are examples of a human resource
information system operating at the :

(a) operational level


(b) management level
(c) knowledge level
(d) strategic level (01)

(xii) Which of the following information is NOT required while computing cost of equity under capital assets
pricing model (CAPM)?
(a) Risk free interest rate
(b) The expected earnings
(c) The beta for the firm
(d) The expected market return (01)
Section B
Question 2:
National Solutions (NS) is a privately owned high technology company established in 2002 by computer engineer, Bill
Gates. It is situated in the country of Redland, a prosperous developed nation with a stable well established political
system. Successive governments in Redland have promoted technology by providing grants and tax incentives. Tax
credits are also provided to offset company investment in research and development. The government, like many
governments worldwide, has invested heavily in a national telecommunications Infrastructure.
However, in 2019 the country suffered an economic downturn that led many companies to postpone technological
investment.
Product areas
In 2019, NS had three distinct product/service areas – data communication components, network management systems
and, finally, technical support.
The international market for data communication components had increased from Rs.3.3 trillion in 2015 to Rs.8.1 trillion
in 2019. Forecasts for 2020 and beyond predict growth from increased sales to currently installed networks. The maturity
of the technology means that product lifecycles are becoming shorter. NS produces components in a relatively prosperous
country where there is significant legislation defining maximum work hours and minimum wage rates. All new
components have to be approved by an appropriate government approval body in each country that NS supplies. This
approval process is both costly and time consuming.
The second product area is network management systems. The success of their product led to it being awarded a
prestigious government technology award for “technological innovation in data communications”. This further enhanced
the company’s reputation. They only have two or three competitors in this specialist market. Unlike component
manufacture, there is no requirement to seek government approval for new network products.

Required
Evaluate the macro-environment of NS using a PEST analysis. (6)

Question 3:
Etihad Limited manufactures and sells three products. The following information is available in this regard:
Year Product -1 (See Product -2 Product -3
note-1) (See note-2) (See note-3)
Sales Market Size Sales Market Size Sales Market Size
Actual ………. Rs. in millions ……….

2017 74 132 3 70 10 100


2018 72 134 2 71 14 120
2019 74 133 2 69 16 140
Forecasted
2020 73 135 2 68 18 160

2021 74 134 1 62 22 190


Note 1: Product 1 is a long-standing product that has been consistently selling in the same market
Note 2: Product 2 has been consistently selling in the same market for quite some time.
Note 3: Product 3 was introduced in 2017 and is still considered new in the market.
In the current year, 2019, the market share of the biggest competitor for Product 1 is 20%,Product 2 is
44%and Product 3 is 37%.

Required:
Analyze the information provided above using BCG matrix and devise strategies for each of three products. (12)
Question 4:
Diamond Investment Limited (DIL) is considering to set-up a plant for the production of a single product X-49. The
details relating to the investment are as under:

(a) The cost of plant amounting to Rs. 160 million would be payable in advance. It includes installation and
commissioning of the plant.

(b) Working capital of Rs. 20 million would be required at the commencement of the commercial operations.

(c) DIL intends to sell X-49 at cost plus 25% (cost does not include depreciation on plant). Sales for the first
year are estimated at Rs. 300 million. The sales quantity would increase at 6% per annum.

(d) The plant would be depreciated at the rate of 20% under the reducing balance method. Tax depreciation is to
be calculated on the same basis. Estimated residual value of the plant at the end of its useful life of four
years would be equal to its carrying value.

(e) Tax rate is 34% and tax is payable in the year the liability arises.

(f) DIL’s cost of capital is 18%. All costs and prices are expected to increase at the rate of 5% per annum.

Required:
Compute the following:
(a) Net present value of the project (10)
(b) Internal rate of return of the project (04)
(Assume that unless otherwise specified, all cash flows would arise at the end of the year.)

Question 5:
A company has just paid an annual dividend of $0.18. Investors expect the annual dividend to grow by 3% each year in
perpetuity, The current share price is $1.55 and the total market value of the company’s shares is $1,200,000.

The company has debt capital on which the yield is 7.8% before tax. The rate of taxis 30%. The total value of the
company’s debt is $350,000.

Required:
Calculate the weighted average cost of capital. Use the dividend growth model to estimate the cost of equity. (06)
Question 6:
a) A UK company expects to pay $750,000 to a supplier in three months’ time. The following exchange rates are
availablefor the dollar against sterling (USD/ GBP):

Spot 1.8570 –1.8580


3 months forward 1.8535 –1.8543

The company is concerned about a possible increase in the value of the dollar during the next three months, and would
like to hedge its FX risk.

Required
Explain how the exposure to currency risk might be hedged, and the amount that the UK company will have to pay in
sterling in three months’ time to settle its liability. (4)

b) A German company expects to receive US$450,000 from a customer in two months’ time. It is concerned about
the riskof a fall in the value of the dollar in the next two months, and would like to hedge the currency risk using
a forward contract.

The following rates are available for the dollar against the euro (USD/EUR):
Spot 1.3015 – 1.3025
2 months forward 25c – 18c premium

Required
Calculate the company’s income in euros from settlement of the forward contract in two months’ time. (4)

(c) A UK company expects to receive $600,000 in six months’ time from a customer. It intends to convert these dollars
into sterling.

The current spot rate for the dollar against sterling (USD/GBP) is 1.8800. The six- month interest rates are 5% per year
for sterling and 3.5% per year for the US dollar.

Required
Show how the company can create a money market hedge for its exposure to receivables in dollar. Also estimate
what the exchange rate should be for a six-month forward contract GBP/USD. (4)
Q.7
The following budgeted statement of profit or loss has been prepared for Quest Company for the four months January
to April for Year 5:
January February March April
Rs000 Rs000 Rs000 Rs000
Sales 60.0 50.0 70.0 60.0
Cost of production 50.0 55.0 32.5 50.0
(Increase)/decrease in inventory (5.0) (17.5) 20.0 (5.0)
Cost of sales (45.0) (37.5) (52.5) (45.0)
Gross profit 15.0 12.5 17.5 15.0
Administration and selling overhead (8.0) (7.5) (8.5) (8.0)
Net profit before interest 7.0 5.0 9.0 7.0

• 40% of the production cost relates to direct materials. Materials are bought in the month prior to the month in
which they are used. Purchases are paid for one month after purchase.
• 30% of the production cost relates to direct labour which is paid for when it is used.
• The remainder of the production cost is production overhead.
• Rs. 5,000 per month is a fixed cost which includes Rs. 3,000 depreciation. Fixed production overhead costs are
paid for when incurred.
• The remaining overhead is variable. The variable production overhead is paid 40% in the month of usage and the
balance one month later. Unpaid variable production overhead at the beginning of January is Rs. 9,000.
• The administration and selling costs are paid quarterly in advance on 1 January, 1 April, 1 July and 1 October.
The amount payable is Rs. 15,000 per quarter.
• All sales are on credit. 20% of receivables are expected to be paid in the month of sale and 80% in the following
month. Unpaid trade receivables at the beginning of January were Rs. 44,000.
• The company intends to purchase capital equipment costing Rs. 30,000 in February which will be payable in
March.
• The bank balance on 1 January Year 5 is expected to be Rs. 5,000 overdrawn.

Required: Complete the cash budget for each of the months January to March Year 5 for Quest Company. (10)
Q.8:
The directors of Noor Packaging Co (NPC), a well-established manufacturer of cardboard boxes, are currently
considering whether to enter the cardboard tube market. Cardboard tubes are purchased by customers whose products
are wound around tubes of various sizes ranging from large tubes on which carpets are wound, to small tubes around
which films and paper products are wound. The cardboard tubes are usually purchased in very large quantities by
customers. On average, the cardboard tubes comprise between 1% and 2% of the total cost of the customers’ finished
product.

The directors have gathered the following information


(i) The cardboard tubes are manufactured on machines which vary in size and speed. The lowest cost
machine is priced at $30,000 and requires only one operative for its operation. A one-day training course is
required in order that an unskilled person can then operate such a machine in an efficient and effective manner.
(ii) The cardboard tubes are made from specially formulated paper which, at times during recent years, has
been in short supply.
(iii) At present, four major manufacturers of cardboard tubes have an aggregate market share of 80%. The
current market leader has a 26% market share. The market shares of the other three major manufacturers,
one of which is JOL Co, are equal in size. The product ranges offered by the four major manufacturers are
similar in terms of size and quality. The market has grown by 2% per annum during recent years.
(iv) A recent report on the activities of a foreign-based multinational company revealed that consideration
was being given to expanding operations in their packaging division overseas. The division possesses large-
scale automated machinery for the manufacture of cardboard tubes of any size.
(v) Another company, Plastic Tubes (PT) produces a narrow, but increasing, range of plastic tubes which are
capable of housing small products such as film and paper-based products. At present, these tubes are on
average 30% more expensive than the equivalent sized cardboard tubes sold in the marketplace.
Required:
Using Porter’s five forces model, assess the attractiveness of the option to enter the market for Card board
tubes as a performance improvement strategy for NPC. (10)

Q.9:
Railway Development Company (RDC) was considering two options for a new railway line connecting two towns. Route
A involved cutting a channel through an area designated as being of special scientific importance because it was one of
a very few suitable feeding grounds for a colony of endangered birds. The birds were considered to be an important
part of the local environment with some potential influences on local ecosystems.
The alternative was Route B which would involve the compulsory purchase and destruction of Eddie Krul's farm. Mr Krul
was a vocal opponent of the Route B plan. He said that he had a right to stay on the land which had been owned by his
family for four generations and which he had developed into a profitable farm. The farm employed a number of local
people whose jobs would be lost if Route B went through the house and land. Mr Krul threatened legal action against
RDC if Route B was chosen.

An independent legal authority has determined that the compulsory purchase price of Mr Krul's farm would be $1
million if Route B was chosen. RDC considered this a material cost, over and above other land costs, because the
projected net present value (NPV) of cash flows over a ten-year period would be $5 million without buying the farm.
This would reduce the NPV by $1 million if Route B was chosen.

The local government authority had given both routes provisional planning permission and offered no opinion of which
it preferred. It supported infrastructure projects such as the new railway line, believing that either route would attract
new income and prosperity to the region. It took the view that as an experienced railway builder, RDC would know best
which to choose and how to evaluate the two options. Because it was very keen to attract the investment, it left the
decision entirely to RDC. RDC selected Route A as the route to build the new line.

A local environmental pressure group, 'Save the Birds', was outraged at the decision to choose Route A. It criticized RDC
and also the local authority for ignoring the sustainability implications of the decision. It accused the company of
profiting at the expense of the environment and threatened to use 'direct action' to disrupt the building of the line
through the birds' feeding ground if Route A went ahead.

Required: Use Tucker's 'five question' model to assess the decision to choose Route A. (10)

Q.10:
Sawari Limited (SL) is engaged in the business of assembling motorcycles. Following IT related matters are under
consideration of the management:

(i) SL uses Inventory Management System (IMS) which is connected with the systems of all its suppliers. IMS generates
and sends purchase orders to the suppliers automatically when the inventory reaches the reorder level. SL has recently
been receiving the complaints of short deliveries. On further inquiry it was revealed that the supplier received different
quantity orders than those actually generated by IMS. Initial investigation revealed that data was changed during
transmission to the suppliers.

(ii) SL’s IT data room maintained at its head office caught fire. All data including last month backup kept within the
premises was lost and critical hardware was also slightly damaged due to this incident. Consequently, SL’s IT operations
suffered a downtime of ten days.

Required: Suggest any three mitigating controls against each of the above matters. (05)
SOLUTION : CAF06 - MFA MOCK (Spring 2022)
Q.1
MCQs
i b
Ii b
Iii c
Iv a
V d
Vi b
Vii c
viii a
ix c
x a
xi d
xii b
Q.2:
Political Factors
 NS is based in a stable, prosperous country, where successive governments have valued and encouraged technology will help
the company to adopt long term strategies and achieve its objective
 Tax incentives and grants are given to companies that invest in technology and in research and development will help in cost
savings.
 Tax credits are also provided to companies that invest in research and development will assist in innovation.
 The government has also promoted the use of technology through a well-publicized awards scheme. NS is a recent beneficiary
of such an award – for “technological innovation in data communications” can be uses as marketing to attract customers.
Economic Factors
The case study suggests that 20X4 saw a downturn in the domestic economy which resulted in a reduction of customer commitment
to long-term investment. Customers may postpone their buying decisions.
Socio-cultural Factors
It appears that electronic communication and information exchange will continue to increase with implications for companies
supplying products and systems to meet these growing needs. All evidence suggests that the social use of services on such networks
will increase.
Technological Factors
Technology is a significant factor in shaping the life cycles of existing products and the introduction of new ones. The technology
sector is extremely innovative, with new and improved technologies constantly emerging. NS must scan the external environment
for such technologies and identify how they might affect the future of their current products.
Legal Factors
NS operates in a country where there are laws defining employer responsibilities and employee rights. It is likely that such regulation
will continue and NS, like all companies working in Redland, have to evaluate the benefits and costs of working within such
constraints.

Q.3:
Product 1: Analysis of Information
 It is a product in a market with low growth.
 Its market share is consistently very high and is forecasted to remain high.
 Its market share in the current year, 2019, is 56% (55.6%), which makes it the market leader.
 It is a long-standing product in the existing market.
 It displays the characteristics of a mature product with dominant position.
 This is a Cash Cow product.
Future Strategies:
 It should be dealt with consistently to keep reaping profits.
 The cash generated by product 1 can be used to invest in other products.
 Market penetration, although difficult to implement in a low growth market, should be used when
expansion is required as it is the least risky approach. For example, this can be obtained through cost
leadership and acquisition of competitors, etc.

Product 2: Analysis of Information


 It is a product in a market with declining growth.
 Its market share is consistently very low and is forecasted to dip lower.
 Its market share in the current year is about 3% (2.9%) which is much less than the 44% market share of
the market leader.
 It is an existing product in the existing market.
 It has weak position and seems to be in the decline phase.
 This is a Dog product.
Future Strategies:
 If the product is generating required ROIs or positive NPV, EM can choose to keep the product till it is
feasible.
 Divest the product as it is projected to decline and may incur losses.
 If EM decides to keep this product for any reason (e.g. profit, brand value, product synergy, etc),
market penetration strategy may be used for expansion, however, EM should avoid risky investments
on this product.

Product 3: Analysis of Information


 It is a product in a market that is showing high growth.
 It has low market share.
 Its market share in the current year is about 11% (11.4%) which is less than the 37% market share of the
market leader.
 It is considered a new product in the new market.
 It resembles an entrepreneurial venture in a high growth market.
 This is a Question Mark product.
Future Strategies:
 The market is growing quickly, there is an opportunity to increase market share, but initially it will
require a substantial investment of cash to increase or even maintain market share. EM should keep
investing in the growth of this product.
 EM can invest cash generated from product 1 into Product 3.
 Differentiation or focus strategy may be used to gain a larger market share.
Q.4:
Diamond Investment Limited
(1) Net Present Value (NPV) of the project
Year 0 Year 1 Year 2 Year 3 Year 4
Cash inflows/(outflows)-Rupees in million
Sales - 300.00 333.90 371.63 413.62
Cost(Sales÷1.25) - (240.00) (267.12) (297.30) (330.90)
Plant depreciation at 25% of WDV - (32.00) (25.60) (20.48) (16.38)
Net profit - 28.00 41.18 53.85 66.34
Tax@34% - (9.52) (14.00) (18.31) (22.56)
Add: back depreciation - 32.00 25.60 20.48 16.38
Cost of plant and its installation (160.00) - - - 65.54
Working capital (20.00) - - - 20.00
Projected cash flows (180.00) 50.48 52.78 56.02 145.70
PVfactorat18% 1.00 0.85 0.72 0.61 0.52
Present value (180.00) 42.91 38.00 34.17 75.76
NPVat18% 10.84
(NPVA)
(2) Internal Rate of Return (IRR) of the project:
PV factor at 22% 1.00 0.82 0.67 0.55 0.45
PV at 22% (Projected cash flow × PV factor) (180.00) 41.39 35.36 30.81 65.57
PV at 22% (NPVB) Rs. (6.87) Million
IRR = Lower rate% + x (high-lower)%
IRR = 18% + [10.84/(10.84 – (-6.87)] x (22% - 18%)
IRR = 20.45%

Q.5:
18 1.03
Cost of equity 0.03
155

= 0.1496 or 14.96%.
350 1,200
WACC 7.8 1 0.30 14.96
1,200 350 1,200 350

= 1.23 + 11.58
= 12.81%.

Q.6:

(a) A hedge against the risk can be obtained by entering into a forward rate agreement to buy $750,000.
The forward rate is the forward rate that favours the bank. This is 1.8535 (and not 1.8543).
The cost of buying the dollars will be $750,000/1.8535 = £404,639.87.
(b) Subtract a premium, add a discount.

Spot rate 1.3025


Premium (0.0018)
Forward rate 1.3007

The $450,000 will be sold in exchange for €345,967.56 (450,000/1.3007).


(c)

The company will receive $600,000 in six months, and will want to receive sterling and pay dollars.
It can do this with a money market hedge by borrowing US dollars now. The interest rate for six months
in dollars is 3.5% × 6/12 = 1.75%. It will need to borrow now:
$600,000/1.0175 = $589,680.59.
It can immediately exchange these dollars into sterling at the spot rate of 1.8800, to obtain:
$589,680.59/1.8800 = £313,659.89

After six months, the dollar loan will be repayable with interest. The total repayment will be $600,000,
and the payment can be made from the $600,000 received from the customer.

The company can do anything with the sterling it receives now from the hedging transaction. If it
chose to invest the cash for six months at 5% per year (2.5% for six months), the investment of
£313,659.89 would increase to:
£313,659.89 × 1.025 = £321,501.39.
To avoid opportunities for arbitrage between the money markets and the forward FX markets, the six-
month forward exchange rate would therefore need to be:
$600,000/£321,501.39 = 1.8662.
Q.7: (a)
SOLUTION:
January February March
Rs Rs Rs
Opening balance (5,000) (14,000) (7,100)
Receipts
Sales (W-1) 56,000 58,000 54,000
Payments
Capital expenditure – – 30,000
Direct materials (W-2) 20,000 22,000 13,000
Direct labour (30 % of production cost) 15,000 16,500 9,750
Fixed production overheads (W-3) 2,000 2,000 2,000
Variable production overheads (W-4) 13,000 10,600 8,800
Admin/selling overhead 15,000 – –
Total outflow (65,000) (51,100) (63,550)
Closing balance (14,000) (7,100) (16,650)
W-1
Cash receipts
Cash from sales Total sales January February March
Rs Rs Rs Rs
Opening 44,000 – –
Receivables
January 60,000 12,000 48,000 –
February 50,000 – 10,000 40,000
March 70,000 – – 14,000
56,000 58,000 54,000
W-2
Payments for materials purchases
December January February March April
Rs Rs Rs Rs Rs
Total cost of production – 50,000 55,000 32,500 50,000
Material cost of production (40%) – 20,000 22,000 13,000 20,000
Purchases made in the month prior to usage 20,000 22,000 13,000 20,000 –
Payments are made in the month following purchase. – 20,000 22,000 13,000 20,000
W-3
Payments for overheads
January February March
Rs Rs Rs
Total cost of production 50,000 55,000 32,500
Overhead cost of production (30%) 15,000 16,500 9,750
Fixed costs (5,000) (5,000) (5,000)
Variable overhead costs 10,000 11,500 4,750
Of the monthly fixed overhead costs of Rs. 5,000, Rs. 3,000 is depreciation which is not a cash expenditure.
Monthly fixed cost cash expenditure is therefore Rs. 2,000.
The opening balance of unpaid variable production overhead cost at the beginning of January is Rs.9,000. This cost
should be paid for in January. Variable overheads are paid 40% in the month of expenditure and 60% the following
month.
W-4
Variable overheads
Cost January February March
Rs Rs Rs Rs
Opening payables for variable overheads 9,000
January 10,000 4,000 6,000 –
February 11,500 – 4,600 6,900
March 4,750 – – 1,900
Total payments 13,000 10,600 8,800
Preparing master budgets
Having prepared budgets for sales and costs, the master budget can be summarised as a statement of profit or loss, a
cash budget (as seen in the previous section) and a statement of financial position as at the end of the budget period.

Q.8:
In order to assess the attractiveness of the option to enter the market for spirally-wound paper tubes, the
directors of NPC could make use of Michael Porter’s ‘five forces model’.
New Entry
In applying this model to the given scenario, one might conclude that the relatively low cost of the machine
together with the fact that an unskilled person would only require one day’s training in order to be able to
operate a machine, constitute relatively low costs of entry to the market. Therefore, one might reasonably
conclude that the threat of new entrants might be high. This is especially the case where the market is highly
fragmented.
Customer Bargaining Power
The fact that products are usually purchased in very large quantities by customers together with the fact that
there is little real difference between the products of alternative suppliers suggests that customer (buyer)
power might well be very high. The fact that the paper tubes on average only comprise between 1% and 2% of
the total cost of the purchaser’s finished product also suggests that buyer power may well be very high.
The fact that a foreign-based multinational company is considering entering this market represents a
significant threat from a potential new entrant as it would appear that the multinational company might well
be able to derive economies of scale from large scale automated machinery and has manufacturing flexibility.
Supply Bargaining Power
The threat from suppliers could be high due to the fact that the specially formulated paper from which the
tubes are made is sometimes in short supply. Hence suppliers might increase their prices with consequential
diminution in gross margin of the firms in the marketplace.
Rivalry
The threat from competitive rivals will be strong as the four major players in the market are of similar size and
that the market is a slow growing market. The market leader currently has 26% of the market and the three
nearest competitors hold approximately 18% of the market.

Substitute Product
The fact that Plastic Tubes (PT) produces a narrow range of plastic tubes constitutes a threat from a substitute
product. This threat will increase if the product range of (PT) is extended and the price of plastic tubes is
reduced.

Q.9:
Is the decision
Profitable
Route A is a profitable choice. Projections demonstrate that its Net Present Value is positive $5 million, $1 million more
than if Route B was chosen. However both routes have additional, uncertain costs which complicate the analysis. With
Route A RDC may be faced with costs arising from direct action by the protest group. With Route B, RDC could have
been faced with the costs of defending a legal action.

Legal
The local government authority has given planning permission for Route A, so RDC seems to have fulfilled its
requirements. RDC does not appear to face the threat of legal action with Route A that it could have faced if it had
chosen Route B.

Fair
RDC has had to decide between the conflicting claims of different stakeholders, the birds versus Eddie Krul and his
workers. Choosing Route A ignores the birds' rights to access their feeding grounds and may threaten their existence.
Choosing Route B means depriving Eddie Krul of a farm that he wants to keep and causing his workers to lose their jobs.
Arguably it has taken the decision not out of an objective assessment of the strength of each claim, but because Route A
is more profitable and it has been influenced by Eddie Krul's threat of legal action.

Right
If a pristine capitalist view is taken that RDC's duty is to choose options that maximise shareholder value, then the choice
of Route A appears to be correct. However, from the viewpoint of a deep ecologist, the decision is wrong because RDC
should not disrupt the existence of other species in any circumstances. From a social and economic viewpoint however,
the decision appears to be right as the line will improve transport and boost economic activity and the construction
work will provide employment opportunities. Route A does not have the adverse consequences of closing the farm that
Route B would have had, including loss of employment and loss of a source of local food.

Sustainable
The decision to choose Route A appears to be the less environmentally-sustainable decision. It could threaten the
existence of an endangered species and disrupt the local ecosystems, with possible unforeseen consequences. Choice of
Route B would though have had implications for social sustainability, with loss of jobs at the farm and adverse impacts
on the local community. Arguably the investment in rail is an investment in a relatively sustainable form of transport
that should reduce the number of cars on the roads.
Summary
Whichever decision RDC chose would have had negative implications for some stakeholders. It chose to prioritise
profitability and minimising the impact on the local community over the effect on the birds and the associated
ecosystems.

Q.10:
(i)
Firewalls to prevent intrusion into the programs that send and receive data.
Restricting access to source data that is transmitted.
Using check sums and check digits to ensure that data received is intact.

(ii)
SL should also store its backup data at some other location.
SL should develop disaster recovery plans, such as an agreement with another entity to make use of its computer
center in the event of a disaster such as a fire or flood.
The company should make suitable maintenance and service agreements with software companies, to provide
‘technical support’ in the event of operating difficulties with the system.
MOCK EXAMS CAF AUTUMN 2022
Company Law
Date: 19th August 2022
3 Hours - 100 Marks
Additional reading time 15 minutes
QUESTION PAPER
Instructions to examinees:
(i) Answer all questions.
(ii) Answer in black pen only.
(iii) Multiple Choice Questions must be answered in answer script only.

SECTION A
Question 1:
Select the most appropriate answer from the options available for each of the following Multiple-
Choice Questions (MCQs). Each MCQ carries ONE mark.
1. In which of the following situations, a company shall be deemed to be unable to pay its debts:

a. if a creditor of Rs. 5 million duly made a demand requiring the company to pay the sum so due and
the company has for 21 days thereafter neglected to pay/secure/compound the sum.
b. if a creditor of Rs. 5 million duly made a demand requiring the company to pay the sum so due and
the company has for 30 days thereafter neglected to pay/secure/compound the sum.
c. if a creditor of Rs. 75,000 duly made a demand requiring the company to pay the sum so due and
the company has for 21 days thereafter neglected to pay/secure/compound the sum.
d. if a creditor of Rs. 75,000 duly made a demand requiring the company to pay the sum so due and
the company has for 30 days thereafter neglected to pay/secure/compound the sum.

2. The registered office of the company is situated in Lahore. The company wants to keep its financial
statement and books and papers at DG Khan. The company may do so by:

a. Passing a Board Resolution and giving a notice of the new address to the Registrar within 7
days of the resolution.
b. Passing an Ordinary Resolution and giving a notice of the new address to the Registrar within 7
days of the resolution
c. Passing a Special Resolution and giving a notice of the new address to the Registrar within 7 days of the
resolution
d. Just giving a notice of the new address to the Registrar within 7 days of the shifting.

3. ABC Private limited, having 5.0 million share capital, wants to enter in a contract with P Limited,
a public company, for purchasing of raw material. One of the Director, Mr. Sham of ABC Private
Limited is also hold 7% share of P Limited. What would be the course of action in the First Board.
Meeting of ABC Private Limited for this Contract:

a. Mr. Sham can Attend and vote in this meeting


b. Mr. Sham cannot vote only, but can attend this Meeting
c. Notice of the meeting shall notice sent to Mr. sham
d. Both a & b
4. Mr. Shoaib is a shareholder of O Limited. Company announces interim dividends and the amount,
which will be paid to Mr. Shoaib is R. 50,000. 2 months ago, Mr. Shoaib purchase stationary products
from the company of Rs. 45,000 on credit and not paid yet. As Per the Companies Act 2017, Total
amount which is to be paid to Mr. Shoaib is Rs. .
a. 45,000
b. 50,000
c. 95,000
d. 5,000

5. Alfa limited, a public company, obtain its certificate of commencement of business on 15 April,
2019. As per the companies Act 2017, it is required to hold statutory meeting up to:
a. 13 October,
b. 18 September
c. 15 August
d. 12 October

6. A company shall give intimation of the payment or satisfaction in full, of any mortgage or
charge to the registrar in the manner specified, within a period of:

a. 15 days from the date of such payment or satisfaction.


b. 30 days from the date of such payment or satisfaction.
c. 30 days from the date of passing the special resolution.
d. 15 days from the date of passing the special resolution.

7. Dwapar equipment finance limited, a non-banking finance company (NBFC), is desirous of


offering secured, redeemable, non-convertible 9% debentures to the public in three or more
tranches over a certain period of time. Which kind of prospectus it is required to issue so that its
purpose is served and there arises no need to take out a fresh prospectus for second and subsequent
offer of securities.

a. Deemed prospectus.
b. Shelf prospectus.
c. Supplement prospectus.
d. Abridged prospectus.

8. An appeal filed by shareholders for challenging the rights, Court will declare the resolution null
and void, if it proved that

a. Certain facts are concealed by company or it is harmful for their interest


b. Appeal file by 10% of the class affected by such resolution
c. Appeal is filed within 30 days
d. All of above
9. ABC, private ltd., with 10 board of directors was served notice of board meeting, 7 days prior to
said meeting on their postal addresses. Mr. M is hospitalized due to some severe illness. Mr. Y
is going to London before board meeting. Mr. X & Mr. B went to Australia for some company
business. Mr. A is busy with his daughter’s marriage & unable to attend meeting. Mr. E’s mother
is hospitalized so he is busy taking care of his mother but he assures to attend meeting via video
conferencing. Mr. D & Mr. P were scheduled to arrive for meeting at 2 pm on same day,
however, flight got delayed by 8 hours. Mr. G & Mr. H are in town & available for scheduled
board meeting. Could board meeting be held?
a. Meeting can be held in evening & director D & director P can join later on
b. Meeting cannot be held because of quorum is incomplete
c. Meeting can be conducted through tele-video conferencing
d. Meeting is postponed due to non-completion of quorum

10. Amar, a promotor for a new proposed company, made an application and got reserved a name for
incorporation of a private limited company but the Registrar of Companies Muridky, before the
incorporation, found that the name was applied by furnishing wrong information.

a. The reserved name shall be cancelled by the Registrar because the name was applied by furnishing
wrong information and Amar who made the application shall be liable to a penalty up to Level 1.
b. The reserved name, after seeking explanation from Amar and after he pays a penalty of Level A
shall be allotted by the Registrar and the company shall be incorporated by this name.
c. The reserved name shall be cancelled by the Registrar but Amar shall not be liable to pay any
penalty because cancellation of name in itself is a penalty.
d. Besides cancellation of the reserved name Amar and Abhilasha shall be debarred from making
an application for reservation of name for one year from the date on which cancellation letter
was issued by the Registrar.

11. A company is said to have been registered when?

a. It files Memorandum of association and Articles of Association


b. It gets incorporation certificate with the Registrar of Companies.
c. It gets certificate for commencement of business.
d. It actually starts its business.
12. In which of the following situations, a contributory shall be entitled to present a petition for winding
up a public company:

I. the number of members is reduced below three.


II. the shares in respect of which he is a contributory have devolved on him through the death
of a former holder.
a. only
b. only
c. Neither (i) nor (ii)
d. Both (i) and (ii)

13. Financial year is:

a. Calendar year
b. Not calendar year
c. Period in respect of which any profit & loss account is prepared
d. Both b & c

14. Copy of register of investments (not in name of the company) will be sent to the members within
Days of request

a. 7
b. 14
c. 21 days
d. Immediately

15. Which of the following officers are mandatorily required to be appointed to manage the affairs of a
listed company under the provisions of the Companies Act, 2017?

a. Chairman, chief executive, company secretary, sole purchase agent


b. Chairman, chief executive, company secretary, share registrar
c. Chairman, chief executive, company secretary, chief financial officer
d. Chairman, chief executive, company secretary, chief financial officer, head of audit
SECTION B
Question 2:

Alpha, Beta and Gamma being the founder of Giga (Pvt) Limited (GPL) got the certificate of
incorporation of the company on 21st March 2019. They have given the undertaking to subscribe
10,000 shares each of Rs 10/per share of the company. As per the provisions of companies Act, 2017
they have to deposit Rs 100,000 /-each on or before 20th April 2019.
Required:
a) Alpha and Gamma deposited Rs 100,000 /each on 15th April 2019 and 18th April 2019
respectively, however Beta could not managed to deposit the amount till 20th April due to
economic crisis. Discuss the implication of companies Act, 2017. (04)
b) GPL approached Billy & Co Charted Accountants (BC) for verification of receipt of the
money so subscribed by its subscribers Alpha and Gamma. The said certificate was issued
by BC on 2nd May 2019.Discuss the implication under provision of companies Act, 2017.
(04)

Question 3:
PZ Mir is running a company of automobile, which deals with luxurious heavy bikes and cars. There
are 2 classes of shareholders in the company PZM limited:

Class A Punjab Shares 85% of total members


Class B Janubi Punjab 15% of total members
In a recent meeting of shareholders to change the rules in the AOA, the shareholders have approved
a special resolution to diminish the rights attaching to ‘Janubi Punjab’ shares. As the special
resolution requires 75% majority therefore shareholders representing ‘Punjab’ shares could easily
pass the special resolution despite opposition of ‘Janubi Punjab’ shareholders.
Required:
You are required to comment on the validity of the above special resolution. (06)

Question 4:
Rapid Constructions Limited (RCL) is a listed company. Advise the Board of Directors of RCL on the
following matters, in the light of the provisions of the Companies Act 2017:
a) A request has been made by Chief Executive, who is also marketing director, for increase in
remuneration of directors from Rs. 25,000 to Rs. 40,000 for attending the Board and sub-
committee meetings and performing extra services. (04)
b) Mr. Dawood, who is a director, wants to appoint his brother in his place as he is going abroad
on vacations. (04)
c) On 5 December 2018 an urgent request has been received from Farid, a member, for
inspection and supply of minutes of the company’s fifteenth extraordinary general meeting
held on 2 December 2018. (04)
d) Junaid was elected as a director of Abid Limited (AL) in its last annual general meeting. After
a few months, it was found that Junaid’s appointment was not valid as votes by certain
members were counted twice due to an error. Few directors have challenged the validity of
all meetings attended by them and the actions taken therein. Under the provision of
Companies Act 2017, discuss how AL can remove Junaid from directorship? (04)
Question 5:
Bhakkar Limited (BL), a listed company, wants to increase its production capacity and is in the
process of acquiring a new plant for its soda ash project. The company is contemplating to finance the
project by issuing ordinary shares to the general public. In order to enhance the credibility of its
expansion project, BL’s management has decided to include a statement from Mr. Shujaabaad, a
mechanical engineer, in its prospectus.
Under the provisions of the Securities Act, 2015 describe the conditions which must be satisfied for the
inclusion of statement from Mr. Shujaabaad in BL’s prospectus. (04)

Question 6:
White Diamond Limited (WDL) was incorporated on 6 February 2021. All the subscribers to the
memorandum of WDL has paid the share money except Aijaz who has subscribed 4,000 shares and is
willing to make the payment before 10 March 2021.
WDL’s management intends to start the operations of the company immediately since minimum
subscription amount is not fixed in WDL’s memorandum and articles of association. Accordingly, for
the purpose of commencement of business, they intend to submit requisite documents with the
registrar. The management is confident that the registrar will accept and register all the documents.

Under the Companies Act, 2017:


a) determine Aijaz’s status in WDL and discuss implications, if any, towards himself as well as WDL,
if he makes the payment before 10 March 2021. (04)
b) explain WDL’s duties if all the subscription money is received on or before 1 March 2021. (04)

Question 7:
A company having seven elected and two nominee directors wanted to appoint an employee of the
Company, Mr. Kay, as Chief Executive of the Company and is desirous to know the answer to the
following before making the appointment:
I. Status of Mr. Kay if appointed as the chief executive,
II. Minimum number of shares to be acquired by Mr. Kay to become Chief Executive.
III. Tenure of office for which Mr. Kay may be appointed as Chief Executive.
IV. How Mr. Kay may be removed?
V. Will it be necessary to obtain members approval for the terms and conditions of Mr. Kay’s
appointment? (06)
Question 8:
The Directors of Generous Limited (GL), a listed company, have recommended dividend equal to
40% of GL’s undistributed profits. However, in the annual general meeting, the shareholders
demanded that 60% of the undistributed profits should be distributed as dividend.
Under the provisions of the Companies Act, 2017 explain the following:
I. whether the shareholders are justified in their demand. (02)
II. whether the directors’ recommendation is appropriate if 70% of the undistributed profits
comprise of unrealized gain on investment property. (03)
III. consequences if the directors fail to pay the dividend within the stipulated time. (03)

Question 9:
Ali is a shareholder of Allied Cement Limited (‘ACL’), a public listed company. ACL has made a
significant profit in the last financial year. The shareholders, including Ali, were expecting a
declaration of a substantial dividend of not less than Rs. 10 per share. However, the board of directors
of ACL proposed a dividend of only Rs. 5 per share. Ali rallied the shareholders and in ACL’s annual
general meeting (held two months ago), the shareholders approved a dividend of Rs. 10 per share.
A few months ago, Ali had written a letter to the company secretary of ACL asking ACL to send his
due dividends to his son, Qadir, in Islamabad. The company secretary is of the view that, since Qadir
is not a shareholder, no dividends can be paid to him.
Due to these issues, the dividend amounts have not been paid and have been lying in ACL’s
bank account for the last two months.
Required:
Under the Companies Act, 2017 advise Ali on the legal consequences of these actions. (06)

Question 10:
a) Akmal Travels (Private) Limited was incorporated on August 1, 2006. In a meeting held on
August 1, 2007, the Chief Executive has informed the directors that due to certain problems
being faced by the company, he is not in a position to hold the first annual general meeting
at an early date.
Identify the latest date by which the directors are required to hold the meeting, giving reasons
with reference to the Companies Act, 2017. (04)

b) A listed company is proposing to hold its annual general meeting on November 20, 1996.
The items on agenda include approval of annual account for the year ended June 30, 1996,
proposed final dividend, appointment of auditors, increase in authorized capital, approval for
investment in associated company.
State how above items should be categorized – ordinary Vs Special business. (06)

c) The Secretary of Badminton (Pvt.) Limited which was incorporated 10 years ago, has
informed you that they are planning to convert their company into a public company. You
are required to advise them on the time period for holding statutory meeting. (03)
Question 11:

a) Abbas limited obtained a loan of Rs. 10 million from Khushal Taraqi Bank. AL provided security
by creating a charge on goodwill. Discuss under Companies Act, 2017 the procedure required to
be followed to get the charge registered with registrar. Also discuss the effect of non-registration.
(06)

b) The Directors of Worldwide Motors Limited (WML) and its subsidiary Worldwide Autos
(Private) Limited (WAPL) have decided in their respective board meetings to shift their
registered offices to Landmark Centre, G.T. Road, Lahore.
The present addresses of the registered offices are:
i. Worldwide Motors Limited, 101, Hali Road, Lahore.
ii. Worldwide Autos (Private) Limited, 2nd Floor, Modern Avenue, Peshawar.
Under the provisions of the Companies Act, 2017 you are required to list the procedures to be
followed by each company for shifting its registered office. (04)

(THE END)
CAF-07 Company Law
Suggested Solution

Answer 1:
1. B
2. A
3. B
4. D
5. D
6. B
7. B
8. D
9. C
10. A
11. B
12. D
13. C
14. A
15. C

Answer 2:

a) In light of the provisions of Companies Act, 2017 Beta 10,000 shares shall be deemed to be
cancelled and his name shall also be removed from the register of members. Moreover, the company
shall have to follow the directions given by the registrar in this respect.

b) GPL shall have to report to the registrar about receipt of subscription money from the subscriber
i.e Alpha and Gamma along will BC certificate on or before 14th April 2020 i.e. within 45 days
from the date of incorporation of GPL.

Answer 3:

As per the requirements of the Companies Act 2017; If alteration affects substantive rights or
liabilities of members or class of members, it shall be carried out only if majority of at least 3/4th of
those affected vote for that (personally or through proxy)

In the current situation, the special resolution relates to diminishing the rights of ‘Janubi Punjab’
shares and is passed with more than 75% majority of members but a separate approval of 3/4th
majority of “Janubi Punjab’ shares have not been taken. Therefore, this special resolution would not
be considered valid.
Answer 4:

a) As per the Companies Act 2017,


“Remuneration of directors for performing extra services (including holding office of chairman) shall be
determined by company in general meeting OR directors (in accordance with articles)
“Chief executive of the company shall be deemed as director”
In the given situation, any increment in remuneration shall be determined by directors in board
meeting
OR members in their general meeting, as defined in articles. However, it should not be above the scale
approved by the RCL.

b) As per the Companies Act 2017,

Any director can appoint alternate directors if following conditions are met:

➢ Approval of board
➢ During his absence from Pakistan of not less than 90 days
➢ He shall vacate office when appointing director returns Pakistan
➢ In the given situation, Mr. Dawood can appoint his brother if above conditions are fulfilled.

c) As per the Companies Act 2017,

“Members can demand certified copy of minutes of general meeting at any time after 7 days of
Meeting (prescribed fee) and company shall provide him within 7 working days of request”.
“Minutes books shall be open to inspection by members for at least 2 hours on each day during the
business hours (without charges), which will be available after 07 days of meeting.”
In the given situation, Farid (a member) can request to the RCL for copy of minutes after 07 days of
meeting by paying the prescribed fee. However, RCL will forward the copy of minutes within 07
days of
application. He can also inspect the minutes during these 02 hours, specified by RCL.

d) As per the Companies Act 2017,

“Members may remove directors through resolution in their meeting. However, the resolution shall
not be considered as passed, if the number of votes casted against resolution (i.e. in favor of director)
is equal to or exceeds:

In case of Elected Directors:

Minimum Number of votes casted at immediately preceding election of directors”.

In the given situation, Junaid can be removed by members through resolution. If Junaid obtain the
minimum votes, casted in election, which is also minimum votes casted to last appointed director,
then he cannot be removed.
Answer 5:
A prospectus inviting persons to subscribe for shares in Bhakkar Limited (BL) shall not include a
statement purporting to be made by an expert (Mr. Shujaabaad), unless he is a person who is not, and
has not been, engaged or interested in the formation or promotion, or in the management, of the
company. Such prospectus including a statement purporting to be made by Mr. Shujaabaad shall not
be issued, unless-
(i) Mr. Shujaabaad has given his written consent to the issue of the prospectus containing his
statement
and has not withdrawn such consent before the delivery of a copy of the prospectus to the registrar for
registration; and
(ii) A statement that he has given and has not withdrawn his consent as aforesaid appears in the
prospectus.

Answer 6
(a)
Relevant Legal Provision:
Every subscriber is required to pay money in cash (against shares subscribed) within 30 days of
incorporation. Further, if no amount is fixed by memorandum or articles, the whole amount of share
capital is minimum subscription.
Decision and Basis of Decision:
If payment is made within 30 days of incorporation (i.e. by 7th March, 2021):
Aijaz’s status will be as of a member of the WDL. There will be no additional implication in this
respect for WDL. If payment is not made within 30 days of incorporation (i.e. by 7th March, 2021):
Shares subscribed by Aijaz shall be cancelled and name of Aijaz shall be removed from register of
members. Company shall follow directions of Registrar in this respect.
Further, company cannot start its operations as minimum subscription is not achieved in this case.
Therefore, shares will have to be subscribed by someone else.

b)

The receipt of money from subscribers shall be reported by WDL to the registrar within forty-five
days from the date of incorporation of WDL (i.e. by 22th March, 2021), accompanied by a
certificate by a practicing chartered accountant or a cost and management accountant verifying
receipt of money.
Further, as WDL is a public company, it will also have to obtain Certificate of Commencement of
Business from Registrar before starting its operations.
Answer 7:
(i) A chief executive is deemed to be a director of the company, even if he was not already a director.

(ii) A chief executive is not required to be a member of the company. Therefore, Mr. Kay
can become chief executive without acquiring any shares.
(iii) Chief executive may be appointed by directors for a period up-to three years (or for shorter
period).

(iv) Chief Executive of a company can be removed before expiry of his term either by:
➢ Company through special resolution, or
➢ Directors with three-fourth majority.
This provision overrides articles or agreement of company with chief executive.
(v) No. Terms and conditions of Mr. Kay’s appointment can be determined by board in accordance
with the provisions in the company's articles.

Answer 8:
(i) Relevant Legal Provision:
Members can only accept, reject or reduce dividend. They cannot increase the amount of dividend
proposed by directors.
Decision and Basis of Decision:
As shareholders have demanded more than divided recommended by directors, their demand is not
justified.

(ii) Relevant Legal Provision:


Dividend shall not be paid out of unrealized gain on investment property.

Decision and Basis of Decision:


In this case, directors’ recommendation is not appropriate. Dividend cannot be recommended out of
unrealized gain on

investment property (being 70% of the undistributed profits). They can recommend up to 30% of
undistributed profits only.

(iii) If dividend is not paid within specified days of its declaration, chief executive of company
shall be:
1. fined for an amount up to five million rupees, and

2. imprisonment for a term up to two years, and

3. ineligible to become a director or chief executive of any company up to a period of five years.
Answer 9:
Approval of dividend:
As per Companies Act, 2017, final dividend is recommended by Directors but approved/declared by
Members in general meeting. However, members cannot approve dividend in excess of amount
recommended by Directors. Therefore, approval of dividend by shareholders in excess of proposed
dividend is not in accordance with law.

Payment of Dividend:
As per Companies Act, 2017, dividend is paid to registered shareholder, or to his order, or to his
bankers, or to a financial institution nominated by him for the purpose. Therefore, view of company
secretary is incorrect. Dividend can be paid to a person who is not shareholder, on the order of the
member. As company has not paid dividend within 30 days of its declaration, chief executive may
be held responsible and may be punishable under the law.

Answer 10:
(a) First AGM is conducted within 16 months from the date of incorporation. Therefore, directors
can hold meeting by November 30, 2007.

Further, company can also apply to Registrar for extension up to 30 days in holding AGM.
(a) Ordinary Business:
➢ approval of annual account
➢ approval of proposed final dividend
➢ Appointment of auditors

Special Business:
➢ increase in authorized capital,
➢ approval for investment in associated company:
(b) A private company which converts itself into public company after one year of incorporation, is
not required to hold statutory meeting.
Answer 11(a):
Under the Companies Act, 2017 a change on goodwill or any intellectual property must be
registered to registrar.
Procedure:
Abbas Limited shall be required to file the specified particulars of the charge together with the copy
of the instrument, if any, verified in the specified manner by which the charge is created with the
registrar for registration.
Time Limit:
Such charge shall be registered within period of 3 days beginning with the day after the date of its
creation
Effect:
If the company or interested persons fails or neglects to register the mortgage or charge as aforesaid,
the mortgage or charge would become void and shall not be accepted as such by the liquidator or
any creditor.
However, this shall not affect any contract or obligation for repayment of the money secured by such
unregistered mortgage or charge.
Answer 11 (b):
(a) Procedure to be followed by Worldwide Motors Limited:

The company will be required to file a notice of shifting the registered office within 15 days of the
change, to the registrar on the prescribed form.
(b) Procedure to be followed by Worldwide Autos (Private) Ltd:

1. Company shall pass Special Resolution to change the place of its registered office.

2. Application shall be filed with Commission to obtain confirmation.

3. In making an order, SECP shall have regard to rights and interests of the members and creditors
of the company (or any class of them).
4. Upon confirmation by Commission, a copy of the order shall be forwarded to Company and
Registrar within 07 days of the order.
5. Within 30 days of the order, Company shall file with Registrar a copy of altered memorandum of
association.
Registrar shall register the same, and shall issue a certificate which shall be conclusive evidence that
all requirements of Act have been complied with.
Certificate in Accounting and Finance Stage Examination
22 August, 2022
3 hours – 100 marks
Additional reading time – 15 minutes

CAF 08 - Audit and Assurance


Mock
Instructions to examinees:
(i) Answer all TWELVE questions.
(ii) Answer in black ballpoint pen only.
(iii) Attempt each part of the question on a new page.

Q.1 (a) List the matters on which an external auditor is required to form an opinion for an unlisted
company, as required by the Companies Act 2017. (05)

(b) During the external audit of Bailey Ltd (Bailey) your firm has concluded that there is a
material uncertainty over whether Bailey is a going concern. State, with reasons, the
implications for the auditor's report. (04)

(c) You are assembling the final audit documentation on the audit of Green Pastures Ltd for
the year ended 30 June 2018.
Required:
List four examples of acceptable changes to the audit documentation during the final assembly
process. (02)

Q.2 Regional Assurance Partners (RAP) is a three-partner audit and assurance firm. The three
partners are Bob, Finlay and Shelley. RAP has been appointed as auditor of Exquisite
Accessories Limited (EAL), a metals processing firm. This is the first year that RAP is
undertaking the EAL audit.

The chief executive officer (CEO) of EAL is Ralph Trinket. Ralph’s wife is a well-known
jewellery designer who frequently produces pieces for fashion shows and other high-profile
media events. Ralph has been vocal about his wife’s successful jewellery business and has
offered to source some exclusive and limited-edition pieces for the audit team.

Owen Jackson, the CFO, and Bob have been friends for many years. Their families usually go
on holiday together once a year, and they frequently socialise with one another. Bob rates
Owen very highly as a CFO and he is confident that Owen will cooperate with the auditors.

Elspeth O’Keefe, a new audit manager at RAP, was previously employed by EAL, and she
prepared the EAL financial report for the period under audit.

Required:
(a) Identify and explain three threats to independence for RAP when performing the EAL
audit.
(b) For each threat identified in (a), state what actions RAP needs to implement to eliminate
or reduce the threat to an acceptable level. (06)
Audit and Assurance | Page 2 of 4

Q.3 You are an audit supervisor of Pluto & Co and are currently planning the audit of your client,
Venus Magnets Co (Venus) which manufactures decorative magnets. Its year end is 31
December 2015 and the forecast profit before tax is $9.6 million.

During the year, the directors reviewed the useful lives and depreciation rates of all classes of
plant and machinery. This resulted in an overall increase in the asset lives and a reduction in
the depreciation charge for the year.

Inventory is held in five warehouses and on 28 and 29 December a full inventory count will be
held with adjustments for movements to the year end. This is due to a lack of available staff on
31 December. In October, there was a fire in one of the warehouses; inventory of $0.9 million
was damaged and this has been written down to its scrap value of $0.2 million. An insurance
claim has been submitted for the difference of $0.7 million. Venus is still waiting to hear from
the insurance company with regards to this claim, but has included the insurance proceeds
within the statement of profit or loss and the statement of financial position.

The finance director has informed the audit manager that the October and November bank
reconciliations each contained unreconciled differences; however, he considers the overall
differences involved to be immaterial.

A directors' bonus scheme was introduced during the year which is based on achieving a target
profit before tax. In order to finalise the bonus figures, the finance director of Venus would
like the audit to commence earlier so that the final results are available earlier this year.

Required:
Describe FIVE audit risks, and explain atleast two audit procedures to address each risk in the
audit of Venus Magnets Co. (10)

Q.4 You are undertaking the audit of Precious Metals Ltd (PML) for the year ended 30 June 2018.
During the audit planning process, the following information is obtained:
(i) PML management’s remuneration is heavily weighted towards incentive-based
payments that rely on optimistic sales targets.
(ii) Recently, the human resources department of PML has been short staffed and has not
been able to provide training to new staff responsible for administrative and financial
processing functions. Generally, new staff members have experience within the
industry.
(iii) One of the outputs produced by PML is a titanium bolt, which is small but very
valuable. There is a high demand for these bolts in the construction industry.
(iv) Receivables are agreed to the sub-ledger, but there is no aging review completed, and
an increasing percentage of total receivables are falling into the 90 days+ category.
Required:
Identify and explain three key fraud risk factors within PML. (06)

Q.5 You are the auditor of Knave Ltd and are in the process of completing the audit for the year
ended 30 June 2018. The following two outstanding matters have been highlighted in your
firm’s completion documentation:
(i) You have heard a rumour that Knave is planning to merge with a competitor, King Ltd. If
accurate, this will have disclosure implications. Management have advised you that although
they have had several meetings with the management of King Ltd, no such merger is currently
planned. Management have offered to make written representations confirming their
intentions.
(ii) The invoices to support the cost of a significant purchase of plant and machinery cannot be
traced. Management have offered to make written representations confirming the cost of the
plant and machinery.
Required:
Are the client’s written representations sufficient to resolve each of the two outstanding
matters noted above? Justify your answer. (04)
Audit and Assurance | Page 3 of 4

Q.6 (a) Briefly define any three examples of data validation controls. (03)
(b) Define intimidation threat. (02)
(c) Differentiate between test data and embedded audit facility. (04)
(d) Explain the terms “control environment” and “control procedures”. (04)
(e) Sampling for tests of controls may lead to two types of incorrect conclusions by the
auditors. Briefly explain these two types of conclusions and their impact on audit. (04)
(f) Give two examples of the situations creating advocacy threat. (02)

Q.7 You are the audit senior responsible for the audit of Spectrum Ltd for the year ended 30 June
2018. During your initial planning meeting with Justin James, the chief financial officer
(CFO), he informs you of the following changes in the company’s operations.

(a) To help achieve budgeted sales for the year, Spectrum is about to introduce bonuses for
sales staff. The bonuses will be an increasing percentage of the gross sales made by each
salesperson above certain monthly targets.

(b) Spectrum plans to close an inefficient factory in country Tasmania before the end of 2018.
It is expected that the redeployment and disposal of the factory assets will not be completed
until the end of the following year. However, Justin is confident that he will be able to
determine reasonably accurate closure provisions.

(c) The chief executive officer (CEO), Geoff Alderton, has just returned from Italy, where he
signed a contract to import a line of clothing that has become the latest fashion fad there.The
company has not previously been engaged in the clothing industry.

Required:
For each of the scenarios provided, outline how the information affects inherent risk. (03)

Q.8 (a) What makes evidence relevant? (03)

(b) Jenny is the statutory auditor of Crystal Ltd, a diamond jewellery shop. Crystal Ltd has
engaged James, an expert, to value its diamond jewellery. While drafting the audit report, she
does not want to take responsibility for the work done by James. She wants to mention the
name of James in her audit report as she is relying on his work. She wants to give a modified
audit report.
Required:
Explain to Jenny the circumstances in which she can give a reference to an expert. (02)

(c) List the contents of typical terms of reference that should be agreed with an auditor's
expert. (02)

Q.9 As a result of work undertaken during the planning stage and audit evidence collected for the
tests of controls stage of the audit, the audit senior, Rose Chung, has determined that there is a
low risk of material misstatement for the following account balances:
(i) telephone
(ii) repairs
(iii) sales commission
(iv) wages and salaries.
Due to the expected reliability of these controls, Rose has undertaken extensive testing of the
controls regarding these account balances, and has concluded that the controls are reliable.
Required:
Identify one audit procedure for each of the income statement account balances above that
will obtain sufficient appropriate audit evidence regarding the accuracy of that account
balance. (04)
Audit and Assurance | Page 4 of 4

Q.10 (a) You are reviewing the audit working papers for Plastics Ltd, which have been prepared by
a junior audit team member. During your review, you noted that when testing the accuracy,
valuation and allocation assertion for the property, plant and equipment (PPE) account
balance, the junior team member tested the PPE additions and disposals during the year and
recalculated the depreciation. No discrepancies were found in the testing. The junior team
member concluded that PPE was appropriately valued. PPE is a material account balance to
Plastics Ltd. (04)

(b) You are the auditor of Critical Solutions Ltd (CSL) for the year ended 30 June 2018.
During your planning process you note that the human resources department of CSL has been
short staffed recently and has not been able to provide training to new staff responsible for
administrative and financial processing functions. Generally, new staff members have
experience within the industry.

While reviewing the accounting system you note that accounts receivable are agreed to the
sub-ledger, but there is no aging review, and an increasing percentage of total receivables are
falling into the 90 days+ category. Time sheets for processing staff are approved by
supervisors, then passed on to Susan Rogers in payroll. Susan prepares the pay sheet
information, which gets reviewed against the time sheets and approved by the CFO, Peter
Cummins, prior to payment being processed.

Access to the information technology (IT) system at CSL is controlled by usernames and
passwords, which are required to be changed regularly through a programmed system prompt.

Required:
Identify and explain two internal control strengths and two internal control weaknesses for
CSL. (04)

Q.11 Electro Ltd is a large Victorian manufacturer of electronic components, which it supplies to
the automobile industry. Electro maintains its inventory records on a computer inventory
system that includes the following details for each product:
 stock code, which includes a physical location code
 product description
 quantity on hand
 unit cost
 total value on hand
 date of last sale
 quantity sold during the year
 sales value of the stock sold during the year.
Required:
List five substantive tests that you could perform using generalised audit software in relation (05)
to the inventory for Electro.

Q.12 (a) Describe the purpose of performing analytical procedures at the planning stage of an audit. (02)
(b) List four indicators of dominant influence being exerted by a related party. (02)
(c) Describe three types of enquiry that an auditor will make of management with regard to
fraud. (03)
(d) Provide three additional audit procedures that the auditor should perform when events or
conditions are identified that call into question the ability of an entity to continue as a going
concern. (04)
(e) Identify four factors that influence sample sizes for tests of controls. (02)
(f) If the evaluation of sample results suggests that there is material misstatement in the
population, what courses of action are available to the auditor? (02)
(g) Identify four requirements of policies and procedures designed to give an audit firm
reasonable assurance that it will be notified of breaches of independence requirements. (02)
(THE END)
CAF 8: Audit & Assurance
Suggested Solution – Mock

Answer # 1

(a) Following are the matters on which an external auditor is required to form an opinion for an
unlisted company, as required by the Companies Act 2017:
1. Whether financial statements conform with the accounting and reporting standards as
applicable in Pakistan and give the information required by the Companies Act, 2017
and give true and fair view;
2. Whether proper books of account have been kept by the company as required by the
Companies Act, 2017;
3. Whether financial statements have been drawn up in conformity with the Companies
Act, 2017, and are in agreement with the books of account and returns;
4. Whether investments made, expenditure incurred and guarantees extended during the
year were for the purpose of the company’s business; and
5. Whether zakat deductible at source under the Zakat and Ushr Ordinance, 1980, was
deducted by the company and deposited in the Central Zakat Fund established under
that Ordinance.

Marking plan: Q.1(a)


1.0 marks for each correct matter.

(b) If directors adequately disclose uncertainty:


 Auditor shall express unmodified opinion on financial statements. (0.5 mark)
 However, auditor shall include “material uncertainty related to going concern” section in
report (0.5 mark) which:
 draws attention to the note (0.5 mark)
 states that a material uncertainty exists (0.5 mark)
 states that auditor’s opinion is not modified in this respect (0.5 mark)

If directors do not disclose uncertainty:


 This will be a misstatement. (0.5 mark)
 Auditor shall express qualified opinion (if effect is material), or adverse opinion (if effect
is pervasive). (1.0 mark)
 Auditor shall also explain the nature of misstatement in “basis for qualified/adverse
opinion” paragraph (0.5 mark) (i.e. material uncertainty exists and financial statements do
not adequately disclose this matter)

Marking plan: Q.1(b)


Available marks = 4.5 ; Maximum marks = 4.0

(c) Examples of acceptable changes to the audit documentation during the final assembly process:
1. Deleting or discarding superseded documentation.
2. Sorting and cross-referencing working papers.
3. Signing off on completion checklists relating to the file assembly process.
4. Documenting audit evidence that auditor has obtained before the date of auditor’s report.

Marking plan: Q.1(c)


0.5 mark for each correct example.

Page 1 of 9
CAF 8: Audit & Assurance
Suggested Solution – Mock

Answer # 2

(a) Threats and Explanation (b) Action RAP needs to implement


Familiarity threat: (0.5 mark) RAP should not make Bob the partner on this
Bob and Owen (the CFO of EAL) are friends. audit. (0.5 mark)
Bob may be too sympathetic towards Owen’s An independent chartered accountant should
interests and too accepting of his work. (0.5 mark) conduct the quality control review of the
engagement. (0.5 mark)
Self-interest threat: (0.5 mark) Gift of jewellery should not be accepted from
Value of jewelleries is likely to be significant. client. (0.5 mark)
Offer to audit team to get jewellery could If any team member has accepted the gift, he
influence the audit team into making the audit should be removed from engagement team. (0.5
‘smooth’ for the client, rather than retaining their mark)
professional scepticism throughout. (0.5 mark)
Self-review threat: (0.5 mark) RAP must remove Elspeth from the audit team.
Elspeth would find it hard to maintain objectivity (0.5 mark)
if she is auditing work that she has prepared. (0.5 If she has already performed some work, her
mark)
work should be reviewed by independent
chartered accountant. (0.5 mark)

Note: Alternate explanations/actions will also be marked, if relevant.

Answer # 3

Audit Risk Procedures


Valuation of Plant and Machinery:  Discuss with the directors the rationale for
Depreciation charge has been reduced after extensions of asset’s lives.
revising life. This reduction may have occurred in  Compare useful life and residual value of
order to achieve profit targets. assets with market.
Existence and Completeness of Inventory:  At year end, auditor shall obtain list of
Inventory count is being conducted at date other inventory adjustments for intervening period,
than year-end. The adjustments may not be made and will corroborate them with other areas
accurately or completely. (e.g. sales and purchases)
 The audit team should increase the extent of
inventory cut-off testing at the year-end.
Valuation of Inventory:  Discuss with management the basis of the
If inventory damaged by fire exists at year-end, it $0.2million scrap value.
may not have been written down to NRV.  Ensure that inventory has been written down
to NRV, and loss charged to income
statement.
Recognition of Insurance Claim:  Read agreement with insurance company to
Insurance claim should not be recognised until evaluate whether this event is covered in
the receipt is virtually certain. There is a risk that agreement.
insurance company may not reimburse full  Check correspondence of management with
amount of the claim. insurance company, and any receipts
subsequent to the year.
 If recovery is not virtually certain,
management should be asked to reverse the
receivable, and disclose as contingency.

Page 2 of 9
CAF 8: Audit & Assurance
Suggested Solution – Mock

Accuracy and Completeness of Bank:  Audit team should remain alert to fraud
Bank reconciliation is not being prepared throughout the audit.
monthly. This may result in errors/omissions,  Management should be asked to prepare
which may become material when aggregated. reconciliation statements (particularly at year
Further, weakness in cash and bank areas indicate end).
risk of fraud.  The reconciling items should be tested in
detail and agreed to supporting
documentation.
Risk at Financial Statements Level:  Throughout the audit, the team will need to
Bonus scheme is based on achieving be alert to this risk and maintain professional
a target profit before tax. Directors can scepticism.
manipulate results to increase the bonus payment.  Any journal adjustments affecting profit
should be tested in detail.
 Detailed review and testing on judgmental
decisions, including treatment of provisions.
Strict Deadline:  Management should be asked to extend the
A reduction in the audit time table will increase time.
detection risk.  If not, auditor should consider to perform
Audit team may not have sufficient time to obtain interim audit.
sufficient appropriate audit evidence. Further,
period to review subsequent events may also be
reduced.

Marking plan: Q.3


1.0 mark for each risk, and 0.5 mark for each procedure; subject to maximum of 10 marks.

Answer # 4

Following are the key fraud risk factors:


1. Management’s remuneration based on optimistic sales levels
Management has an incentive to overstate sales to achieve optimistic performance targets and
receive their bonus.
2. Small, high-value inventory
Titanium bolts are small but very valuable and so could be stolen by staff for resale in market.
3. Increase in overdue debtors
There is a risk that cashier may be receiving cash from debtors, and not recording it.

Marking plan: Q.4


2.0 marks for identification and explanation of each fraud risk factor.

Answer # 5

Rumours that Knave is planning to merge with King:


1. As this matter relates to management intention, a client representation letter is appropriate to
resolve this matter. (1.0 marks)
2. However, auditor should also obtain minutes of board meeting to corroborate this matter. (1.0 marks)

Page 3 of 9
CAF 8: Audit & Assurance
Suggested Solution – Mock

Missing invoices relating to a significant purchase of plant and machinery:


1. A client representation letter is not appropriate evidence because other evidence should be
available that shows that Knave purchased the plant and machinery. (1.0 marks)
2. Other evidence that could be obtained includes:
i. a copy of the purchase invoice from the supplier (0.5 mark)
ii. written confirmation from the supplier that Knave purchased the plant and machinery.
(0.5 mark)

Answer # 6

(a) (any three of following)


1. Limit Test/Check:
A check to ensure that a financial value does not exceed some predetermined value.
2. Range/Reasonableness Test:
A check to ensure that a numerical value does not fall outside the predetermined range of
values e.g. wages of employees fall within 10,000 to 25,000.
3. Sequence Test:
A check to ensure that all entries in system are in numerical sequence e.g. there is no
missing purchase invoice.
4. Existence Test:
A check to ensure that a code/number exists by looking up the code in the valid record
e.g. whether a supplier exists.
5. Format/Field Test:
A check to ensure that format of a data in a field is either alphabet or numeric or
alphanumeric e.g. that there are no alphabets in a sales amount field.
6. Check-digit:
A check-digit is a digit that is calculated in a mathematical way from the original code
and then is added to the end of the code as extra-digit e.g. to detect transposition errors.
7. Batch Total:
A Batch total is the sum of number or value of transactions which are entered into the
system in batch. A manually calculated sum from source documents is compared with
electronically calculated sum from recorded transactions to ensure transactions are
accurately and completely recorded.
Marking plan: Q.6(a)
1.0 marks for each example, subject to maximum of 3.0 marks.

(b) Intimidation Threat:


Threat that an assurance team member is deterred from acting objectively because of threats,
undue influence or pressure. (1.0 mark)
Examples:
1. Threat of dismissal of auditor (or his relative) by client from current or proposed
engagement. (0.5 mark)
2. Threat of litigation by client. (0.5 mark)

(c) Test Data:


Test data is a set of dummy transactions developed by auditor and processed by client’s IT
system. After processing, auditor compares actual results with expected results to determine
whether controls are operating effectively. (2.0 marks)

Page 4 of 9
CAF 8: Audit & Assurance
Suggested Solution – Mock

Embedded Audit Facilities:


It is auditor’s computer programs that is built into the client’s IT system to allow the audit to
carry out tests at the time transactions are processed in ‘real time’. In this approach, a dummy
department is built into client’s accounting system that operates every time the ‘live’ process is
run. (2.0 marks)

(d) Control Environment:


Control environment includes attitude, awareness and actions of TCWG and management
regarding entity’s internal control and its importance in the entity. (1.0 marks)

In evaluating the control environment, auditor considers the following matters:


 Audit committee and board of directors have significant influence in the organization
and actively participate in business.
 Management actions and attitudes show character, integrity, and ethics.
 Management is committed towards Competence.
 No tolerance over code of conduct (e.g. petty theft)
 Management's operating style and philosophy is not aggressive towards financial
reporting.
 Organizational structure is appropriate according to business.
 Management assigns authority and responsibility appropriately.
 Human resource policies emphasize on strong control environment.
(0.5 marks for each matter, subject to maximum of 1.0 marks)

Control Procedures:
Control procedures are the policies and procedures (other than control environment) to ensure
that entity’s objectives are achieved. (1.0 marks)

In evaluating the control activates, auditor considers the following categories:


 Authorization Controls
 Physical Controls
 Segregation of duties
 Controls over using Information Processing
 Reconciliations
 Performance Reviews/ Management Controls
(0.5 mark for each category, subject to maximum of 1.0 mark)

(e) Risk of Incorrect Rejection/Rejection of Right:


Erroneous Conclusion:
Auditor concludes that controls are weak (but in fact they are strong) (1.0 mark)
Effect on Audit:
It affects efficiency leading to increased procedures. (1.0 mark)

Risk of Incorrect Acceptance/Acceptance of Wrong:


Erroneous Conclusion:
Auditor concludes that controls are strong (but in fact they are weak) (1.0 mark)
Effect on Audit:
It affects effectiveness leading to inappropriate opinion (1.0 mark)

Page 5 of 9
CAF 8: Audit & Assurance
Suggested Solution – Mock

(f) 1. Firm promoting shares of an assurance client.


2. Acting as an advocate of assurance client in litigations or disputes (e.g. tax disputes) with
third parties.
3. Commenting publically on future events.
Marking plan: Q.6(f)
1.0 marks for each example, subject to maximum of 2.0 marks.

Answer # 7

(a) Inherent risk is increased.


Tying bonuses to reported sales increases the risk that sales staff will process fictitious sales,
make sales to uncreditworthy customers or manipulate sales between periods to increase sales
and thereby their bonuses.

(b) Inherent risk is increased.


The CFO will be required to make a subjective estimate of the provision for compensation
payable to employees.
Further, assets held for same may not be appropriately classified and measured.

(c) Inherent risk is increased.


The company is entering into a market in which it has no experience. The product is susceptible
to fashion changes which increases risks for its valuation at year end.
Further, the fact that the product is imported also exposes the company to foreign exchange
fluctuations.

Answer # 8
(a) Relevance:
Relevance deals with the logical connection between audit evidence/procedure and assertion
being tested.
Examples:
1. If auditor wants to test Occurrence (i.e. overstatement), relevant procedure will be to test
recorded amounts.
2. However, if auditor wants to test Completeness (i.e. understatement), relevant procedure
will be to test information outside accounting system
3. A procedure relevant for one assertion may not be relevant for other e.g. inspection of
tangible assets may provide evidence for Existence, but not for Rights & Obligation.
4. Tests of controls are designed to evaluate the operating effectiveness of controls. Relevant
evidence would be conditions which indicate performance or deviation of control.
5. Substantive procedures are designed to detect misstatements in financial statements.
Relevant evidence would be conditions which indicate misstatement.
Marking plan: Q.8(a)
1.0 marks for definition, and 1.0 marks for each correct example; subject to maximum of 3.0
marks.

(b) Auditor shall not refer to the work of expert in auditor’s report unless:
1. it is required by Law or Regulation (1.0 marks), or
2. such reference is relevant to understand nature of modification to the auditor’s opinion.
In such circumstances, the auditor may need the permission of the auditor’s expert before
making such a reference (1.0 marks).

Page 6 of 9
CAF 8: Audit & Assurance
Suggested Solution – Mock

(c) The auditor shall agree, in writing when appropriate, on the following matters with the auditor’s
expert:
(i) The nature, scope and objectives of that expert’s work;
(ii) The respective roles and responsibilities of the auditor and that expert;
(iii) The nature, timing and extent of communication between the auditor and that expert,
including the form of any report to be provided by that expert; and
(iv) The need for the auditor’s expert to observe confidentiality requirements.
(0.5 marks for each matter)

Answer # 9

Account Balance Audit Procedure


(i) Telephone Inspect telephone bills.
(ii) Repairs Inspect invoices from professionals providing repair services.
(iii) Sales commission Perform analytical procedures to confirm reasonableness of sales
commission. (or recalculate sales commission on sample basis)
(iv) Wages and salaries Perform analytical procedures to confirm reasonableness of wages and
salaries.
Also, recalculate salaries from payroll sheet on sample basis.

Marking plan:
1.0 marks for correct and relevant procedure in each area.

Answer # 10

(a) The junior audit team member’s conclusion is not correct (01 mark). Sufficient appropriate audit
evidence has not been obtained in relation to the accuracy, valuation and allocation assertion (01
mark)
. Further procedures will be needed to specifically cover the value of existing assets (01 mark); for
example, testing for indicators of impairment (01 mark).

(b) Internal Control Strengths:


1. IT system is controlled by usernames and passwords, which are required to be changed
through a programmed system prompt (01 mark)
2. Timesheets are authorised by supervisors and checked against the paysheet and
authorised by the CFO, Peter Cummins, before payments are processed (01 mark)

Internal Control Weaknesses:


1. Lack of training for new staff in administrative and financial processing areas (01 mark)
2. Receivables ageing is not reviewed regularly (01 mark)

Page 7 of 9
CAF 8: Audit & Assurance
Suggested Solution – Mock

Answer # 11
Possible substantive tests of inventory using CAATs would include the following:
1. The inventory list could be totalled and recalculated to check the mathematical accuracy.
2. Samples could be obtained for testing. Samples could include both items selected on a materiality
basis and a random basis.
3. Product wise analysis of sales can be performed to identify items that have not been sold in
recent months and may have become obsolete.
4. An NRV test (i.e. last sales price compared with the unit cost of the item) could be completed.
5. A number of other exception reports could be generated; for example,
a. where an item number appears twice,
b. where there are negative quantities or
c. where the recorded unit costs appear unusual.

Marking plan:
1.0 mark for correct and relevant procedure involving CAAT. Other suitable procedures will also be
marked as correct.

Answer # 12

(a)  To obtain understanding of entity and its environment, (1.0 mark) and
 To assess risk of material misstatement, so that procedures can be increased on high risk
areas (and vice-versa). (1.0 mark)

(b)  Founder of entity who is also currently managing the entity.


 Significant transactions referred for final approval.
 Vetoed significant business decisions.
 Little or no debate on proposals by related party.
 Transactions involving the related party are not reviewed and approved.
(0.5 mark for each indicator, subject to maximum of 2.0 marks)

(c) ISAs require the auditor to make inquiries of management in respect of:
i. their process in place for identifying and responding to the risks of fraud.
ii. their assessment of the risk of fraud.
iii. any specific risks of fraud identified or likely to exist.
iv. any communications within the entity in respect of fraud (e.g. code of conduct).
(1.0 mark for each inquiry, subject to maximum of 3.0 marks)

(d) 1. Evaluating management’s plans for future action, and whether it is feasible. Obtain
representation from management/TCWG regarding future plans and their feasibility.
2. If management has prepared a cash flow forecast, evaluate:
a. Data used is reliable., and
b. Assumptions used are adequately supportable.
3. Consider effects of subsequent event on going concern assessment.
4. Read:
a. Minutes of the meeting of shareholders/directors regarding financial difficulties.
b. Loan agreements, and compliance with their terms.
c. Latest available financial information.
5. Inquire legal counsel regarding existence of litigations and claims.

Page 8 of 9
CAF 8: Audit & Assurance
Suggested Solution – Mock

6. Confirm existence and adequacy of borrowing facilities (e.g. support from


directors/holding company).
(01 mark for each procedure, subject to maximum of 04 marks)

(e) 1. Tolerable rate of deviation


2. Expected Rate of deviation
3. Desired Level of Assurance
4. Extent to which auditor plans to rely on controls
5. Population Size
(0.5 mark for each factor, subject to maximum of 02 marks)

(f) Sample has not provided a reasonable basis for conclusion about population that has been tested
(1.0 marks)
. Auditor shall perform further substantive procedures (1.0 marks).

(g) 1. Internal Quality Control Review of engagements


2. Discussing ethical issues with TCWG.
3. Appointment of ethics partner.
4. Declaration of independence to be signed by all team members at start of audit.
5. Policies and procedures to identify and communicate interests or relationships between
engagement team and client identified during audit.
(01 mark for each procedure, subject to maximum of 02 marks)

(THE END)

Page 9 of 9

You might also like